Vous êtes sur la page 1sur 66

Law of Lawyering Class Notes When reading, the main thing to identify in a problem is the issue and find

which rule(s) (and comments) are relevant. PL: Almost all of our problems will require you to apply one or more of the Model Rules and related comments. There will be some other doctrines in play as well, but mostly it is the MRPC. Ill be looking for citations to the relevant rules and comments on the exam. 1) The exam will consist of problems like the ones that are in the book, but likely shorter. 2) ABA Model Rules of Professional Conduct are effectively the law for this class and for the MPRE. A) Attorneys are licensed by states, and so state ethics rules apply. They are not exactly the same as the model rules. B) The comments to these rules are very important. Remember to read those along with the rules. What subjects you to professional discipline? Read ALL of Rule 8.4 (it is not just violating the rules). The Case of the Innocent Lifer (pg. 29) 1) Alton Logan was convicted of murder. Two attorneys (not representing Logan) found out that one of their clients, Andrew Wilson, admitted to killing the victim when asked. The attorneys have an attorneyclient relationshi with Wilson. However, they know Logan is innocent since Wilson is the real murderer. They wish to reveal the information. 2) What type of information is this? It is privileged and confidential. A) Privileged Information Privilege is a concept from the law of evidence. Privilege means that the information is not admissible unless the person holding the privilege waives it. B) Confidential Information Confidentiality is a concept from the law of agency. PL: Agency law does concern confidentiality, but the best way to think about confidentiality for this class is to see it as arising from Rule 1.6. i) ABA Model Rule 1.6(a) A lawyer shall not reveal information relating to the representation of a client unless the client gives informed consent, the disclosure is impliedly authorized in order to carry out the representation or the disclosure is permitted by paragraph (b).

Law of Lawyering Class Notes ii) What if the information is revealed from a prospective client that you have not formally created an attorney-client relationship with? It is also likely confidential and protected. Well come back to this. PL: Yes, we will see this in connection with Rule 1.18 later. 3) We have two problems in this case. A) We have confidential information under Rule 1.6(a). What can you do? i) Exceptions to Confidentiality a) Rule 1.6(a) Informed Consent ABA Model Rule 1.0(e) Informed consent denotes the agreement by a person to a proposed course of conduct after the lawyer has communicated adequate information and explanation about the material risks of and reasonably available alternatives to the proposed course of conduct. PL: One thing I wanted you to recognize from this part of the problem is that some terms in the Model Rules are defined terms. Be aware of which ones have definitions in Rule 1.0. In Logans case, you can have a conversation with the client to get them to waive confidentiality. PL: As we saw in class, this conversation must include the necessary information to lead to informed consent. disclosure is impliedly authorized in order to carry out the representation The disclosure must be for the benefit of the client. Implied Authorization is defined in Comment 5 to Rule 1.6. PL: Comment 5 is useful here to see the kinds of things that constitute implied authorization. Here, think about things that are for the benefit of the client and are routine (like sharing info with partners). In this case, there is no implied authorization to reveal this information to anyone.

Law of Lawyering Class Notes b) Rule 1.6(b) A lawyer may reveal information relating to the representation of a client to the extent the lawyer reasonably believes necessary: 1) to prevent reasonably certain death or substantial bodily harm; If Logan had been sentenced to death, then this exception would work. You could argue that physical (and potentially mental) bodily harm is possible while in prison. However, the harm must be reasonably certain. Also, you can only reveal it to prevent such harm. The information is still privileged. If Wilson does not waive the privilege and no one believes you, then your information does nothing. PL: In other words, it will prevent nothing, and so 1.6(b)(1) would not be available if you know that revelation will have no practical effect. 2) to prevent the client from committing a crime or fraud that is reasonably certain to result in a substantial injury to the financial interests or property of another and in furtherance of which the client has used or is using the lawyers services; This is to prevent the commission of a crime. PL: or a fraud; note the requirement that the lawyers services have been used; cf: Alls Not Well. 3) to prevent, mitigate or rectify substantial injury to the financial interests or property of another that is reasonably certain to result or has resulted from the clients commission of a crime or fraud in furtherance of which the client has used the lawyers services; This is to prevent financial injury. PL: This is to prevent, mitigate or rectify injury from the commission of a criminal or fraudulent act that has already happened. 4) to secure legal advice about the lawyers compliance with these Rules; Does not apply in Logans case. 5) to establish a claim or defense on behalf of the lawyer in a controversy between the lawyer and the client, to establish a defense to a criminal charge or civil claim against the 3

Law of Lawyering Class Notes lawyer based upon conduct in which the client was involved, or to respond to allegations in any proceeding concerning the lawyers representation of the client; or Does not apply in Logans case. 6) to comply with other law or a court order Does not apply in Logans Case. Note: None of these rules REQUIRE an attorney to reveal the information. There are other rules that do compel an attorney to reveal certain information, but Rule 1.6 does not. PL: For example, we will see that 4.1(b), 3.3(a)(3) and 3.3(b) require revelation of confidential information in certain circumstances. ii) You can also just reveal the information and violate the rule. What happens then? a) ABA Model Rule 8.4(a) It is professional misconduct to violate the Rules of Professional Conduct b) The information still ends up being privileged and might not help Logan. B) If you get around confidentiality, this is privileged information under the law of evidence. This must also be waived. 4) Wilson said the information could be released after he died. However, in general, the duty of confidentiality continues after the client-lawyer relationship has terminated essentially forever. (See Comment 18 to Rule 1.6) 5) In this case, the attorneys properly followed the rules and did not reveal the information while Wilson was alive. My Client is HIV Positive (pg. 30) 1) The lawyer is representing Ken, who has been arrested for a bar fight. The lawyer asks Ken to give him any information that will make him more sympathetic to help reduce the bail. Ken tells the lawyer that he is HIV positive, but instructs the lawyer not to tell the judge if Anna might find out since he does not want Anna to know. Ken also says that he takes precautions but the lawyer does not believe Ken is a cautious man. The lawyer wants to reveal this information to Anna. 4

Law of Lawyering Class Notes 2) Does the lawyer have an attorney-client relationship with Ken? A) In order for there to be an attorney-client relationship, there must be a mutual understanding that such a relationship exists and the client reasonably believes that the attorney is the clients attorney. PL: Usually, an attorney-client relationship will obviously exist, as a result of mutual assent. Frequently there will be a written engagement agreement, but that is not necessary. The purpose of this discussion is to warn you against the possibility of creating an attorney-client relationship unintentionally. If there are circumstances as a result of which the client reasonably believes there is such a relationship, regardless of what the lawyer thinks, the relationship exists. Advice and assistance are the most frequent circumstances that courts and juries find give rise to such a reasonable belief. i) What can give rise to an attorney-client relationship? Advice or assistance that will give a client a reasonable belief that the person giving the advice or assistance is acting as an attorney. Do not do things for anyone that a lawyer would do for their client unless you want to be liable for those actions as a lawyer. a) Solution: I am not your lawyer. (Disclaim the fact so the client does not believe you are their lawyer.) ii) Remember, the doctrine is the reasonable belief of the client. If you have represented someone in the past, in order to indicate to the client that you are no longer their attorney, tell them you are not their lawyer. B) Here, there is an attorney-client relationship with Ken. Is there one with Anna? i) Anna was a former client. We are presuming no attorney-client relationship still exists with Anna. PL: Although the problem does not raise this issue, we used it as an occasion to think about the episodic client who may think a lawyer is still representing her, much like a patient may say Yes, I have a dentist, even though no procedure is actually underway. ii) Anna is paying the bill. This does not change the attorney-client relationship with Ken and it does not create an attorney-client relationship with Anna. See Rule 1.8(f) 5

Law of Lawyering Class Notes 3) What type of information is at issue here? A) This is confidential and privileged information. How can you get around Rule 1.6? i) Informed Consent appeal to Kens better nature! (If you love her, you will tell her.) PL: Remember also you have to talk about the advantages and disadvantages of revealing the information. This is what makes consent informed. ii) to prevent reasonably certain death or substantial bodily harm a) This one can be argued. If she is already infected, it may become AIDS. If she is not already infected, she will become infected. If Ken is using precautions, then you have to make a credibility judgment. There may even be potential harm to Annas child. All of this has to be reasonably certain. iii) You will not be punished for not revealing this information. a) Some states have said that there are some circumstances where you HAVE to act, but the ABA has not. B) Privilege is not a concern in getting the information to Anna since it is not going before a court. Alls Not Well (pg. 31) 1) Ben, the lawyer, helps with real estate transactions in a resort town. He helps a family sell a beautiful resort home. Several days later, he goes to a barbeque and runs into the engineer who did a report on the home. The engineer asks Ben about a well problem at the home. Ben is unaware of this problem. The engineer sends Ben the report with the well problem. Ben compares this report to the report he received and that was sent to the buyer. The report he had and that was sent to the buyer did not include the information about the well. What does Ben do now? The sale might have been fraudulent since this information was omitted from the report provided by Bens client. However, Ben did not know of this admission. 2) Fraud is an intentional act, so Ben has not committed a criminal act since he did not know that any information was omitted. 3) What can you do? What steps can you take? 6

Law of Lawyering Class Notes A) Ask the client if they can explain the discrepancy and provide the hard evidence to show them the omission. PL: This is an important step. You suspect but do not know that this was fraud. If it was an accident, and you accuse your client of fraud, you have created a new and different problem. B) You have to determine whether or not there is a crime or fraud in the first place. The buyer might have been negligent. The buyer might have no basis to show they reasonably relied on the information if no information was provided about the well and the buyer did not ask about the well. C) If there is a crime or fraud, 1.6(b)(2) (prevent commission of a crime PL: or fraud) and (b)(3) (prevent financial injury PL: As in this problem, where the fraud has already happened bu the injury has not) may be available to the attorney. D) Regardless of the clients knowledge, advise the client that they should come forward with the information to protect them from additional liability and then get informed consent to inform the buyer. E) 1.6(b)(3) gives a pretty clear cut answer here that you can reveal this information to the buyer. PL: Remember that this is permissive, but in this case it is very much in your interest to reveal. The problem will surface, and you will be much less of a target if you reveal sooner rather than later. 4) Unlike the previous problems, this was information that did not come from the client. It came from the engineer. A) Thus, this information is confidential. However, it is not privileged since it is not between you and your client. Slip and Fall (pg. 39) PL: The point of this problem at this point in the sequence is probably not apparent. The main thing to realize is that there are different versions of privilege as applied to corporate agents (compare the control group test with the Restatement test). It is important for the companys lawyer to know which version applies in his or her jurisdiction, so that as conversations occur the lawyer exercises 7

Law of Lawyering Class Notes more or less caution, depending upon whether the privilege applies. It is important for Ediths lawyer to know, for reasons that will be apparent soon. That lawyer must be careful not to invade the corporations privilege. Stay tuned for Slip and Fall Redux. 1) What this is about initially (well come back to it later) is privilege in the context of a corporate client. If you have a corporate client, what communications to whom are privileged. 2) Edith went to a department store. She fell. She broke her hip. She sued saying the floor was excessively waxed. The General Counsels office of the corporation dispatches an investigator to interview people about the accident. The investigator writes up the interviews and memos about the conversation. A lawsuit arises and there is a request for production for these documents that were produced by the investigator. This is clearly work product. 3) As an attorney, the company is your client. You deal with many people including employees, managers, directors, etc. However, these people are not your clients. The company is your client. PL: See 1.13(a) 4) Do you want it to be privileged? Yes, this gives you control to reveal good information and hide bad information. A) Privilege does not cover the underlying facts, just the communications made to the lawyer. i) Thus, opposing party could ask an employee what happened, but cannot ask what the employee said to the lawyer. 5) The information is protected by work product doctrine. The work product doctrine can be overcome if there is a substantial need and there is no other way to get the information. 6) If the information is privileged, then the other side cannot overcome privilege and get the information. A) Get the information privileged! What is the doctrine of privilege? See Rule 1.13 first, and also case law. i) Upjohn Co. v. United States only control group communication is privileged ii) Restatement of the Law Governing Lawyers 3rd 73 pg. 48 Under the restatement, privilege for purposes of organizations applies to a communication between the agent of an organization 8

Law of Lawyering Class Notes and a lawyer (or lawyers agent) and the communication concerns a legal matter of interest to the organization. 7) Are any of these reports privileged? A) Max Burkow, head of maintenance i) Upjohn and the restatement apply B) Tim Morse, last waxed the floor i) The restatement applies C) Tina Sandstrom, a salesperson in mens furnishings i) The restatement applies D) Rex McCormick, a buyer in the rug department, who worked at Tracys but was off that day i) The restatement applies E) Delia Corcoran , retired head of maintenance who established store standards F) Ed Rivera, president of company that supplies wax to Tracys G) Angie Kuhl, a buyer in the watch department In a Box 1) Martin has a client that does general corporate work and has a partner. Martins partner is Sally. Sally also has a client. Sally is representing Jennie. Jennie wants to do an investment with a joint venture, which includes Endicott and BIG MONEY! The firm does not represent Endicott. Martin represents Font & Blue. While representing F&B, Martin learns that the attorney general is investigating Endicott and that Endicott might not be trustworthy to do a deal with. 2) Hypo: If Sally read about the investigation in the WSJ, would she tell her client? Does she have to? A) She probably would tell her client. Under Rule 1.4(b), she HAS to tell the client. PL: This is the duty of communication. It will be in play frequently as we talk about what course of action lawyers must take in particular circumstances. 3) Martin knows. Sally does not know. Jennie does not know. What is the issue here? Why cant Martin tell Sally? 9

Law of Lawyering Class Notes A) Martin found out this information representing Font & Blue and it is confidential information. It is not implied authorized to tell his partner because it is not for the benefit of Font & Blue. What can Martin do to make sure that he can reveal the information to Sally to protect Sallys client Jennie from the potentially bad deal with Endicott.

Notes for LOL 5-26-11


In a Box pg. 84: Sally (Jenny) and Martin (Font & Blue) are partners. Jenny is thinking about doing deal with Endicott. Martin knows Endicott is being investigated. What type of information is this? Confidential (*Remember to distinguish from Privilege) Is there a duty of communication? Sally does not have the information Martin does. Under what circumstances can Martin tell Sally? o Informed consent, Implied authorization, or Exceptions under 1.6(b) (financial loss, death or substantial bodily injury, etc.) o Alarm PL: The alarm actually should go off later. See below. (whoop whoop whoop): Sally can tell Martin info under 1.6(a) implied authorization PL: This is the classic example of when a clients information may be shared by implied authorization: one lawyer to another in the firm, for the purpose of helping the client. On the other hand, if Martin purported to share F & Bs information with Sally under implied authorization, there would be a problem because the sharing is being done not to help F & B but rather to help a different client. o 1.6(b) to comply with other lawthere may be some securities law problems here. PL: If other law permitted or required disclosure, then the problem is solved. Assume there is no such law. o To get informed consent from Font & Blue: Make an inquiry to find out the same information from somewhere else? No way to get F&Bs INFORMED consent because you cant tell any specifics. You may obtain Consent but it wont be INFORMED. PL: This is actually the place for the alarm. As we tried to articulate the conversation seeking informed consent, Jennys confidential information was being conveyed without her permission or other authorization. o Is there any way to legitimately obtain informed consent from Jenny to tell Font & Blue? No, its the same problem as above. o Suggest to the Attorney General to make the investigation Public. PL: Yes, but there is no reason for the AG to do that at this point. o Change in Problem: This is a foreshadowing of a problem we will do later*** o Suppose its not two lawyers, Its only One lawyer Martin: o He would violate the rules of conduct to reveal to Jenny. PL: 1.6 o 1.4(b) Obligation to reveal PL: to Jenny and Obligation to be silent PL: to F & B simultaneously 10

Law of Lawyering Class Notes o Find a way to talk Jenny out of it PL: If this suggestion is about giving advice not to do the deal without revealing the information, it does not solve the communication problem. Furthermore, the information might not matter to Jenny, and if that is the case then you are talking her out of the deal when she would choose to proceed if she was adequately informed. o 1.16(a) DECLINING OR TERMINATING REPRESENTATION (Withdrawal) o If you are in a position that in order to continue the representation you will have to violate the MRPC (Model Rules of Professional Conduct) you MUST withdraw (Martin cannot represent both) o Could withdraw from F&B and then PL: continue to represent Jenny? NO PL: remember, the duty of confidentiality survives the termination of the attorney client relationship. o Could he withdraw from Jenny and then represent F&B? yes, the duty of communication, unlike the duty of confidentiality, does not survive the severance PL: termination of representation o NOTE: You can also analyze this as a conflict of interest problem (MRPC 1.7) PL: We will see this later. Once you know about Rule 1.7, come back to this problem and analyze it through that lens. Surprise Discussion: Nickols v. Keller You are hired to represent a client who has a Workers Comp claim. Follow rules of Civil Procedure. Elements of claim: Someone was hurt at work while performing the duties. No proof of fault needed. We are representing Longan in a workers comp case. Longan was working with a Lathe machine and was injured. Do we have a duty to tell him he may have a cause of action in Tort? (MRPC 1.4 Duty to communicate) (a)(2) Clients objectives: Clients objective is to receive as much as possible which would require a tort action. Yes there is a duty to speak. What if the K said the lawyers representation is limited to just the Workers Comp claim? (MRPC 1.2 (c) Scope) Lawyer may reasonably limit the scope with clients informed consent. If you dont speak, you may have a malpractice suit if you dont tell and the statute of limitations runs. PL: One additional reason to mention this problem is to introduce you to Rule 1.2(c) and the circumstances under which you can limit the scope of your representation. You may just spend 5 mins. explaining the possibility. *The duty to speak is not as obvious to notice as you may think. PL: Read the Nichols case. You will see how the court arrived at this conclusion, with which I agree. Remember that the duty of communication is about fulfilling reasonable client expectations. Here surely the client would reasonably expect the lawyer who is, after all, the expert about the law hired to help the client obtain the maximum relief permitted by the law to at least tell the client of additional means of obtaining relief. See Comment 5 to Rule 1.4: The guiding principle is that the lawyer should fulfill reasonable client expectations for information consistent with the duty to act in the client's best interests, and the client's overall requirements as to the character of representation. Suppose you have an insurance defense lawyer, someone has a cause a death in an auto accident. Lawyer is hired and paid by the insurance company. Who is the lawyers client? Technically, the client is the driver. 11

Law of Lawyering Class Notes (May also represent the insurance Co.) PL: We will study the insurance defense triangle formed by the relationships among the lawyer, the insured and the insurance company later in the semester. In the course of representation, the lawyer learns of circumstances that the driver has claim against the insurance Co for bad faith. Is the lawyer limited in some way? 1.2 (c) Lawyer is sure to make a limitation explicitly. Does the lawyer have a duty to speak about the bad faith claim against the insurance co? Duty to speak and should be silent. Conflict should be recognized for how complex the problems like this can get and intertwine. GO GET SOME GUIDANCE. PL: I mention this circumstance really just to illustrate how complex the duty of communication can be. It is usually straightforward, but there are advanced problems that are more complex. What are the odds pg. 85 Client wants to make a decision about a business transaction. How do you verbalize the risks? Should you give numbers? Can it even be quantified? Low, Moderate, High? (Language of Prediction) Example: Longan said there is a 0% chance that you will win against meThose people won on Summary Judgment later that day. PL: Thanks for including this embarrassing example in the class notes. True story. Harm: Client may take it as a guarantee. It puts an undue weight on the client. Its an illusion. Gives a false sense of precision. **More comfortable talking in language of prediction.** PL: Emphasis here on the danger of apparent precision. The client is entitled to your advice and your prediction of risk, but the advice can actually be misleading if it is conveyed with false precision. The inherent and unpredictable uncertainty is better conveyed with less precise language, such as low risk, moderate risk, high risk. PL: Here, be sure to think about Rule 2.1, which we discussed later in class. You are to render candid advice, but that candid advice might actually be misleading if it is too precise. To that extent, this is a lesson in your professional responsibility as well as a lesson about counseling. This is much less of a lesson in ethics but a lesson in Counseling your client. Ms. Niceperson pg.87 1980-Longans first year of Law School. (There will be a time in which you can rely on your opponents incompetency) PL: The quotation is from the orientation speech I heard as a brand new law student on September 30, 1980. Floyd Abrams said, You will be amazed throughout your career at how often you can rely upon the incompetence of opposing counsel. After already asking for a extension, the opposing lawyer calls and asks for more time on a stipulation filing. You agree. The agreement, however, is irrelevant because the time will run out if he doesnt file by the deadline. Do I tell him? **This is the toughest problem of professionalism** PL: By this I mean that this SET of problems, which pit the duty to client success against duties to others, including the system of justice, raise difficult and fundamental issues. When CAN you rely upon the incompetence of opposing counsel, and when must you or may you do something about it? This is a means to an end NOT and objective matter. PL: See Rule 1.2(a) and the distinction between client objectives (belong to client) and means (shared responsibility). 1. Tell client. (1 vote) Why talk to client? Let them know what is going on. Make them aware. Duty of conscience: personal morals. My reputation will suffer: Seems to put lawyers interests in front of clients interests. Longan thinks you should talk to the clienthere is the situation, here is how it happened. Here are the consequences. Here are the choices. *What if Client says NO and you still want to tell? Most of the time the client will do what you suggest. When you cant agreethere is no rule that says the tie goes to the lawyer (look at MRPC 12

Law of Lawyering Class Notes 1.2 Comment (2)) Seek a mutually acceptable resolution or say the 5 magic words (I am Not your lawyer) or the client says (Youre Fired). *If youve been fired, you can do nothing or you could tell the other lawyer (then you get sued for breach of fiduciary duty) **1.16 (a)(3) must withdraw when fired subject to 1.16(c) When other attorney gets notice he will ask why. To tell is a breach of confidentiality. **1.16(b)(4) You may withdraw if the client insists on taking actions repugnant to the lawyer 2. Do Nothing. (3 votes): Its a dog-eat-dog world. To engage in deceit is misconduct. PL: I did not mean to suggest that a failure to tell the other lawyer is deceit or is otherwise misconduct. I do believe that there is a duty to discuss first with the client. Your advice, and the ultimate course of action, may ultimately be not to tell. Why not talk to your client? Is there a duty to tell? 3. Call the other attorney. (Most of the votes): DO NOT TELL THE CLIENT Text Longan(the lawyer) WTF. PL: This is the scenario in which you do not discuss the situation with the client but instead just inform the other lawyer of what he needs to do. Then the client finds out that you had the chance to ensure victory and did not even tell the client. The CLASS (not me) suggested the substance of the clients text message but that is absolutely the reaction I wanted you to recognize and fear. Substantive Justice is not being served. PL: An underlying question is whether you bear any responsibility to ensure substantive justice. The usual answer is no you have a role to play and as long as you play that role within the rules then substantive justice is not your responsibility. Rhetorical question: should it be? Guidance: (MRPC 1.2 (a)) Lawyer shall abide by the clients decision on objective matters, settlement, (in criminal case) guilty plea, jury waiver, defendants testifying. Lawyers must consult with the client about the means. (1.4(a)(2)) Comment 6: Many routine matters are limited. Handle this on the front end. Explain that lawyers cooperate on routine things like extensions. Get the clients consent then. PL: This is about routine matters like cooperation on scheduling. The particular problem is not one you could anticipate, but you can anticipate in every case the likelihood of a need to cooperate with opposing counsel.

Sidebar: Suppose youre at trial. Opponents questioning the witness (Objection hearsay) Is this a means? But you would not consult hear because it is not reasonable PL: See Rule 1.4(a)(2) A Lawyer shall: (2) reasonably consult with the client about the means by which the client's objectives are to be accomplished; Note also comment [3] Paragraph (a)(2) requires the lawyer to reasonably consult with the client about the means to be used to accomplish the client's objectives. In some situations depending on both the importance of the action under consideration and the feasibility of consulting with the client this duty will require consultation prior to taking action. In other circumstances, such as during a trial when an immediate decision must be made, the exigency of the situation may require the lawyer to act without prior consultation. In such cases the lawyer must nonetheless act reasonably to inform the client of actions the lawyer has taken on the client's behalf. I Dont Plea Bargainpg. 88 Can he do this? 13

Law of Lawyering Class Notes

Is this a means or an End? A means. MRPC (1.2(c)) I will be your lawyer BUT (limits the scope of representation) Why would you do this? Truly innocent, Message to prosecution, Gambler, etc. Duty to Communicate: Plea bargains are the autonomy of the client (rule 1.4) Comment 2a Lawyer who receives a plea bargain or offer of settlement MUST tell the client. Can client refuse to hear a plea bargain? No because its not INFORMED consent. PL: This question concerns whether the client can in advance cut off the lawyers duty of communication by agreement. There are limited circumstances under which the client can do so, such as a decision that the client does not need to be informed about a settlement offer in a particular amount, or a plea bargain with particular conditions. See Comment 2 to Rule 1.4: a lawyer who receives from opposing counsel an offer of settlement in a civil controversy or a proffered plea bargain in a criminal case must promptly inform the client of its substance unless the client has previously indicated that the proposal will be acceptable or unacceptable or has authorized the lawyer to accept or to reject the offer. See Rule 1.2(a). If circumstances change, however such as the development or revelation of evidence that may change the lawyers or clients evaluation of the particular offer then the duty to communicate the unacceptable offer will re-emerge. Surprise Problem: Suppose criminal defense lawyer represents a defendant who has been convicted and is on appeal. Def. is sophisticated. Lawyer while working on the case spots a number of problems. Decide which points you want to raise on appeal? You dont want to raise 20 points.you must keep focus. Lawyer wants to pick 4 strongest ones. Does the lawyer have to discuss with client? Is this a means? Yes. 1.2(A) 1. Make decision on 4 points first then talk to client. What if client then says is that all youve got? Then you say what all the arguments are. Then client says I dont want 1-4 I want 1,7,12. There has been a consultation, There has been a disagreement, you can: (1) agree to do what client wants. PL: As with other means to an end that concern evaluation of the strength of legal arguments, you can assume that the client will usually agree with you. If not, then perhaps you will yield to the clients informed choice. If you cannot agree, then the result is either withdrawal or being fired. Suppose client is NOT sophisticated. Client is indigent. There are 20 possible arguments. 4 are good. Consult with client by mail. Client likes 1, 7, 12, 18 but wants you to raise ALL claims (120). PL: Read Jones v. Barnes and compare its result with what you know about Rule 1.2. I believe the two are inconsistent and that the Supreme Court has adopted a patronizing view of the role of the lawyer, one that it would not have adopted for a paying, sophisticated client. Of course, my opinion on the result does not matter, except that it is an occasion to reinforce for you the usual rule on means/ends and the role of the client. Indigent clients still have autonomy. You DONT have to do what the client says. The rule is only about consultation. 14

Law of Lawyering Class Notes *If client gets appeal to Habeas and the client sues(1.6(b)(5)) Can talk about confidential communications to defend yourself against claims of malpractice. **Read Jones v. Barnes: Its up to the lawyer to decide with indigent clients. Id Rather Die pg. 95 Convicted murderer has had 4 stays of execution. Lawyer is convinced that the client should continue to appeal but client says Do NOT appeal. What do you do? 1.2(A) Objectives belong to the client 1. Could say Im going to fight this regardless 2. Explain to court that clients objectives have changed and you have been instructed to stop proceeding 3. Withdraw because youve been fired **Lara needs to stay quiet** PL: ??? CLIENT AUTONOMY NOT lawyer autonomyClients get to decide AFTER your advice and consultation. PL: One point to remember about this problem is that the client decides the objectives of the representation. If the objective is to hasten execution, and that conclusion is reached after consultation and by a competent adult, then the lawyer either assists or refuses to assist. The lawyer is not to substitute the lawyers objective (e.g., such as political opposition to the death penalty) for the clients objective. Accept the Offer pg 96. Divorce case. Chloe is ending the marriage and feels guilty about it. Russell makes a low-ball offer Lawyer for Chloe says she shouldnt take the offer because you can get more. Chloe wants to take it. **1.4(b) Communication of offer** Make sure they are thinking about it. Give client time to think it over before signing. Counseling needs to occur about economics, not legal stuff. MRPC 2.1 The lawyer as ADVISOR: give CANDID advice, independent professional judgment. Comment 1: should not be deterred from giving candid advice because it will be unpalatable. May refer to other things besides the law. *Still be tactful and kind.* *Not enough to say you must consult under 1.4(b). 2.1 helps fill in the conversation.* Would/could you withdraw? PL: Remember 1.16(b)(4): perhaps the client wants to engage in a course of conduct with which you have a fundamental disagreement. You would then have the option to withdraw, subject to 1.16(c). It is also possible that a sincere statement that you will withdraw rather than help with this agreement will help convey how strongly you feel that it is a mistake. If you do this, I cannot be your lawyer. Engage in self-preservationCYAIf you let her do what she wants. Have her sign something that says I have advised you against this. (this is MORE effective) PL: Perhaps more effective as a means of conveying how much you believe this is a mistake. We will start with one you have NOT seen. Read rule 1.14 Clients who lack mental capacity. 15

Law of Lawyering Class Notes

Taking a quiz to start the class o Dont tell my wife Duty of communication exists if an attorney-client relationship exists Material information relating to the outcome of the representation Assets could likely be affected by this information Potential problems Conflict of interest Confidentiality Obliged to tell, obliged not to tell How solve? 2.1 candid advice Advise the husband to tell the wife 8.4 misconduct Cannot be deceitful 1.16 withdraw from representation Probably could only withdraw from the wife in order to represent the husband, otherwise could not represent either Make sure you take a moment to decide how to handle it Divide the legal duties Lawyer can represent the husband in the separate matter but needs to have a new lawyer represent the couple in regards to the will But 1.2 scope of representation Lawyer shall not counsel a client to engage in conduct that a lawyer knows if fraudulent PL: This would require some research. Is what the husband doing fraudulent? He is making at least an implicit representation to his wife, that she will inherit all of his assets if he dies first. He is doing so for the purpose of inducing her to execute a will leaving him all of her assets, and the representation is untrue. She is relying on it. If under the law this is fraud, then the lawyer cannot assist with it. o For next class, submit DVD player instructions to Longan PL: Thanks o Confusion 1.14 client with diminished capacity Permitted under 1.6(a) to disclose information that would further the interests of the client if the client falls under the criteria set forth in 1.14(b) How determine whether the client falls under (a) or (b)? Doesnt this require some sort of medical determination? Comment says an attorney may seek guidance from a professional who can make a diagnosis PL: So disclosure as necessary to decide if protective action under 1.14(b) is an option is impliedly authorized under 1.6(a). Lucid moments intermingled with moments of clearly diminished capacity Seems unwilling to settle at some times and confused about settling at others Perhaps lawyer should come back another day, at a time when it is likely the client will be more able to make a decision PL: If this works, then it is nothing more than the lawyer doing what is necessary to have as normal an attorney-client relationship as possible. See 1.14(a). o Break Confusion conclusion Determine that a risk exists May take action at that point 1.16 withdraw Client fails to fulfill an obligation to the lawyer Other good cause PL Best fit, in my view.

16

Law of Lawyering Class Notes


Maybe the representation has been rendered unreasonably difficult based on actions undertaken by the client 1.4 communication Share information necessary for the adequate representation of the client 1.14 diminished capacity Share information necessary for the adequate representation of the client Furthers the clients interests or otherwise reasonably protects them Best solution is durable power of attorney May not be possible PL: Because if there is not one already in place, she may not be capable of giving one. Go to the court to seek appointment of a conservator or guardian ad litem PL: An option, and probably the best one here. Must be able to demonstrate the client's diminished capacity Could represent both the nieces and the client PL: Possibly. You would need to do a conflict analysis under 1.7, about which we will learn more. For the moment, it is important to understand that it is not automatically forbidden. The interests of the guardian and the ward may be aligned. It is acting in the best interests of the client So you could tell the nieces what they need to know to protect her interests They could go to get the appointments necessary Need to get help, but from whom? Diagnostician Court Family Ultimately figure out the course of action based on 1.14 options Slip and Fall Redux o 4.2 communication with other attorney's clients Shall not communicate with said person unless other lawyer provides consent Prevents abuse by attorneys that would otherwise take advantage of clients What if they call you? Instruct them that the client may not call you without the lawyer's consent and the conversation is over and cannot take place without the attorney giving consent Terminate the conversation immediately Document the call and any future calls as well as the response made (as it should be an appropriate response under the MRPC) File deposition requests PL: I mentioned depositions just to remind you that you CAN talk to a represented party; Rule 4.2 prevents you from doing so informally without the permission of the other lawyer, but you can nevertheless use a deposition to get the information you seek. Client is the entity So can you talk to the employees? Depends on the test used PL: Be careful here. The test for whether you can speak to them is Comment 7, Rule 4.2. The test for whether they are covered by the privilege is different and can vary by jurisdiction. I would have to give you the prevailing view on corporate privilege on an exam. Are agents/employees the client? There are two extremes: they either are or are not the clients But this is not the rule You can oftentimes talk to these people without the other attorney's permission Comment 7 to 4.2 This is likely to be on the exam Client is Constituent of the entity who

17

Law of Lawyering Class Notes


Supervises, directs, or regularly consults with the entity's attorney concerning the matter; Has authority to obligate the organization with respect to the matter; or Whose act or omission in connection with the matter may be imputed to the organization for the purposes of liability (who is the face of the entity insofar as this matter is concerned?) PL: When I mentioned the face of the entity, I was referring to the first of these three categories. But remember the categories, not my characterization. How do you know beforehand who the client is? PL: Again, be careful. It is not that these individuals are clients. They are not necessarily clients. The client is the entity. The individuals in these three categories may be protected by Rule 4.2 without being clients. The real question here is, how do you know before speaking to them that they are in one of the three categories? You may not. You could try to explain away an otherwise improper contact by saying that you did not know you were in violation, but that is not a position you want to be in if you can help it. Motion to disqualify if the attorney does in fact meet with the "clients" without the permission of their attorney 8.5 disciplinary authority PL: I think you mean 8.4(a), for violation of a rule of conduct. Court has inherent implied power to impose sanctions simply by the existence of that court Court responsible for protecting the fairness of the proceedings How know its unfair? Look to MRPC, if it says you shouldnt do it, then doing it is unfair See Chambers v. NASCO, USSC case where justices imposed a super totes ridic ducats sanction on attorneys PL: Super totes ridic ducats? $996,000 Rule 4.2 says that the attorney cannot talk to clients that the attorney KNOWS are represented by other counsel Max? No PL: Category 3 Tim? No PL: Category 3 Rex? Yes PL: None of three categories Delia? Yes Can talk to former constituents because they are no longer represented by the attorney PL: They may never have been personally represented by an attorney. The main point here is that former constituents are fair game, although (see below) you still have to be careful not to invade the corporations privilege. But they may have individual representation, so that would reset the entire issue under 4.2 What can you ask Rex? "What did you tell your lawyer?" creates a privilege issue Upjohn test permits it Control group test permits it Restatement does not permit it First question should be "Are you represented by counsel?" Ensure that you abide by 4.2 4.3 unrepresented person PL: If they are fair game, you still owe them duties as unrepresented persons, under Rule 4.3. The only advice you can give, when acting on behalf of a client, is that the person should seek legal counsel 4.4 respect for rights of third persons Violated when lawyers ask questions that invade the privilege

Break

18

Law of Lawyering Class Notes


Permitted to communicate under 4.2 as long as the conversation stays within the contours of the privilege between client and attorney

Housing Discrimination PSA o Lawyers cannot get others to break the rules o 8.4 misconduct PL: through the acts of another 8.4(a) o 5.3 supervisory duties of attorney o 4.2 communication Exceptions Authorized by law Attorneys can basically run stings like the police do based on caselaw See, e.g., Hill v. Shell Oil Co. Videotaping and surreptitious recordings have been permitted Public policy - serves justice and the greater good PL: The authorized by law exception is more firmly established for police undercover investigations than for civil testers, but the two issues raise many of the same issues. Authorized by court order Authorized by attorney's consent o 4.1 truthfulness Cannot knowingly make a false statement of material fact to another during the course of representation of a client Facebook? o Think about divorce cases o Say to paralegal "friend the wife" Is that ok? Class remains undecided Look to bar association opinions regarding these new avenues of discovery and technology Perhaps 4.4(b) requires you to say "I got some damning information about your client from facebook. They probably didnt mean to let us have it. But we got it. How ya like me now?!?!?!" PL: Look at 4.4(b). It is about information inadvertently sent, not information carelessly left available and discovered by another party. o The MRPC is being forced into situations for which they were not written 4.4(b) used to be about faxes o Longan will provide us with a bar association ethics opinion regarding facebook PL: Will do. Thanks for the reminder. Watch course web page. Something you should know o Former constituent Thus, can discuss, generally speaking You can discuss, but you can't necessarily "see" what they have or obtain certain documents and other materials 4.2 comment 7 o 4.4(a) may be at issue if you gather certain information If material is privileged than attorney cannot obtain the information o There is a duty to seek out your clients best interests - due diligence o Authenticity issues The paralegal could be a tester and not a former constituent PL: I would be careful about calling this person a tester, although I see the analogy. Think about the danger that the whole thing is a setup to try to bait you into a 4.4(a) violation, for the purpose of getting you disqualified. Stranger things have happened. Be careful. The material is asserted as discoverable but it possibly might not be and could be in fact privileged o "Having all the tools doesnt mean you can build the bookcase" PL: Remember that I said this in the context of congratulating you on acquiring all the tools 4.2, corporate privilege, 4.4(a), 4.3, among others. Creativity and ingenuity coupled with experience are important to figure these situations out o Hire a lawyer

19

Law of Lawyering Class Notes


Deals with the snitch Can look at the info and decide whether or not to pass it along Ensures due diligence for the client If material is discoverable, then it looks like everything is good and you've even slammed the other side for failure to obey discovery requests

Practice Exam Questions o 1. Elderly client Can you talk to a geriatrics doc Yes, under Rule 1.14 Comment 6 saying that you can speak with a diagnostician This is the decision to speak to the diagnostician Look at Rule 1.6(a) The information about what you have seen and heard of your client that makes you think she might have diminished capacity is confidential, it is related to the representation. You can still speak to the doctor though because the communication is authorized by implied authorization under 1.6(a) b/c it is for the benefit of your client. PL: As I mentioned in class, this became a part of the comment to 1.14 after the ABA issued an ethics opinion that stated that such a revelation was impliedly authorized by 1.6(a). o 2. Slip and Fall Can your partner talk to a witness of the fall who works for Chess Store? Chess Store is represented by counsel You are governed by the Restatement of the Law Governing Lawyers definition of the scope of the corporate attorney-client privilege The restatement is going to give the privilege to the corporation if the convo is between the corp.'s attorney and one of the corp.'s agents and the convo is dealing with a legal matter important to the corporation. Rule 4.2 Comment 7 The corporation is who is represented the witness is an agent and some are covered and some are not covered. This witness is outside the prohibition of 4.2 but the conversations are about the fall and that is a legal matter of interest to the corporation so your partner can't talk to the witness about the conversations. o NOTE: Understand the differences between the scopes of corporate privilege PL: But remember I will give you which version is in effect.

Unethical Fees o Rule 1.5 (a): basically it can't be unreasonable. (c) special section governing contingent fees (d)(1): no contingent fees for representing clients in divorces There is a public policy incentive to allow for reconciliation and in favor of marriage and if the attorney's fee is contingent upon divorce then the attorney's might be counter to reconciliation and also might promote discord in order to maximize the sum agreed upon so that the attorney will be well paid. (d)(2): no contingent fees for representing a criminal defendant 20

Law of Lawyering Class Notes Comment explains that the lawyers incentives might be skewed if their payment is dependant upon an acquittal. The attorney might push for trial as opposed to negotiating a plea bargain or mitigation. (e): dealing with the division of fees between lawyers who are not in the same firm. There will be an old exam question brought relating to this PL: I will find one. Thanks for the reminder.

Rule 1.5 o What gets lawyers into the most trouble? Whether or not the fee is reasonable Lawyers are routinely discipline for fees being unreasonable PL: under one particular circumstance, described below The easiest way to get disciplined is when a lawyer takes a few, does nothing and then refuses to return the fee Ex: I pay you a fee to file a guardianship petition and you dont file then refuse to give it back Other than those situations in which lawyers take fees and do nothing, lawyers are very rarely sanction for their fees being unreasonable NO MATTER HOW YOU STRUCTURE THE MONEY, FEES CREATE INCENTIVES IN THE LAWYERS (it is your job to resist the perverse incentives) o What are you worth? (pg. 151) Is this simply a question of contract law? The class baulks at the idea that Scheck was out of line charging Porgby half a billion. However, the MRPC is an overlay on K law. Looking at Rule 1.5 Scheck is a lawyer and is bound by the rules of conduct and can only charge a reasonable fee This is a deeper analysis than K law b/c there is a separate body that evaluates the situation Can it be used to justify this fee? 1.5(a)(1)-(8) is kind of an objective analysis of the situation and using that objective stance does the Bar determine it to be reasonable. (a)(3) is a problem creator. Rule 1.5 is very paternalistic, it goes behind the attorney after the fact. In fact, in a way it takes the ability of the client to contract with the attorney away from the client. It is also a consumer protection mechanism The Bar wants to ensure that lawyers are not taking advantage of clients o HYPO: Ill represent you (criminal defense) but my minimum fee is $10,000 up front. I bill by the hour which will being after we meet the 10K You are low hanging fruit my friend PL: I made this comment in the context of the hypothetical. We supposed the defendant had been charged with drug trafficking but only had $10,000. One of you said, He must be a bad drug dealer. A low-level drug dealer who can be arrested at little risk to law enforcement is low-hanging fruiteasy to acquire. Is there anything wrong with this? This is not a deposit, this is a no matter what jump of point In this situation, the clients right to terminate representation is chilled because theyve already paid a nonrefundable minimum fee 21

Law of Lawyering Class Notes Maybe you cant hire anyone else One of the doctrines we rely on is that lawyers can be fired at any time and anything that interferes with that is looked on with much suspicion PL: As a result, one of the types of fee arrangements that is given special scrutiny is the nonrefundable fee when charged to a client who may have no other funds, especially if the lawyer reserves the right to charge MORE than the initial nonrefundable fee. o Vocabulary Flat Fee Nonrefundable minimum fee General retainer Special retainer o 1.5 is one of the nitty gritty details that fits into the broad category of fidelity to the client o The client gets to decide to settle under 1.2(a) o Meet Sara Bennet (pg. 161) What can you pay a lawyer for? Usually you pay the lawyer for a lawyer time or for performing a particular task. In this situation, Sara is being paid to be available. She's being paid to hold a place for the client in the case that they might need her. The client is paying a special retainer A special retainer: a fee that retains an attorney or firm to handle only a specific circumstance/situation. A general retainer: a fee that retains an attorney or firm's services in whatever matters might come up for the client. PL: For some particular period of time This representation might create conflicts of interest for Sara. And for her entire 700-lawyer firm There doesn't seem to be a reason to paternalistically prohibit general and special retainers because they have value to both the client and the firm. It's a known established income for the firm and it gives the client the peace of mind that they are represented. o The coercive nature of a nonrefundable minimum fee doesn't really fit in anywhere in Rule 1.5. You have to remember though that the 1.5 list of factors to be considered for determining reasonableness is not exhaustive. There is very real suspicion of nonrefundable fees; it causes the courts the most trouble. On the other hand, flat fees are a little different because it doesn't put the client at a distinct disadvantage. In a nonrefundable fee situation the client is at risk of incurring more fees depending on the amount of work the attorney does and in a flat fee scenario, the money paid is the end of the story. Under 1.16(d) the client can get the flat fee back if they fire the attorney before the task is completed. Minimum Fee Schedules o Petra Bento's Hybrid Fee Agreement (pg. 175) Contingent fees Basically, if you settle against my advice then the fee agreement changes and you owe me an hourly rate or a percentage but if you take my advice then the fee remains contingent. Is this practice okay? Is this any different than saying to the client "you can't settle without my consent", which you can't do FYI PL: Georgia courts have said that clients may not make this deal. Nope this is an illegal contract and the courts are not gonna allow it to happen. o Surprise Problem Dealing with Contingent Fees (pg. 229) 22

Law of Lawyering Class Notes Mary O'Meara story Lawyer says the claim is worth 1million and it'll take 2 years to take to trial and it's in the interest of the defense to drag it out. Problem here: it's an important case but in the meantime Mary has to eat Can she get welfare, disability, food stamps, government assistance Lets say she can't or it's not enough, what do you do while she's in dire financial straights? The insurance co. of course will offer an extremely low settlement that is way less than the value of her case, what do you do? There are companies that provide "bridge loans" (basically a pay day loan operation) PL: I described it as a pay day loan operation because in my experience the amount charged for interest and risk is quite high. Mr. Dorer stated that he had seen some that were reasonable, and for those my characterization would be unfair. As her attorney you have to do the math and figure out what is in her best interest financially. Rule 1.8 is a conflict of interest rule in particular the conflict between the interests of the client and the attorney. (conflicts of interest are the heart of this course) On the exam, you will see a question(s) that says you are on your firms conflict of interest committeehandle this problem It's a series of very specific circumstances 1.8(e) establishes that the lawyer SHALL NOT provide financial assistance to a client in connection with pending litigation with certain expenses. The rule used to say that the attorney could not make the court costs and expenses contingent and so lawyers would just ignore routinely when their client lost b/c they absolutely were not going to go after the client for the costs and expenses. The rule was changed b/c of the practice so now costs and expenses can be made contingent upon the outcome of the matter. PL: It is untenable for the bar to have a rule that is widely and openly ignored and not enforced. Here, the rule caught up with practice, much as the rule against other financial assistance to clients may catch up with the apparently common practice of providing such assistance. Attorneys can flat out pay the costs and expenses on behalf of an indigent client. Is there a jurisdiction that permits some kind of financial assistance or that flat out omits 1.8(e) from its actual rules? Longan thinks it might be D.C. PL: It is. See p. 231 Lawyers routinely loan the money in this situation despite what the rule says. The history for justifying 1.8(e) is that there is a bias against litigation and this rule would keep clients from bringing suits that might not otherwise be brought. This is not the way our society thinks anymore. It is absolutely against the rules and it is probably another circumstance in which the rule is going to change to fit the practice of attorneys. This is problematic b/c how can we say we have rules which govern us but only when we agree with them. Client-Lawyer Conflicts 23

Law of Lawyering Class Notes


o o

When does the conflict of interest problem arise? Be sensitive to that and how do I handle it when it arises? Lawyer. Realtor. Any Problem? (pg. 220) In a routine real estate transaction, the attorney represents the lender. What is it about the lawyer fulfilling this two roles that causes problems? PL: raemember in the problem the lawyer is not representing the lender. The lawyer is representing a party to the transaction as a broker and that same party as a lawyer. A broker's interest is in making sure the deal closes and the lawyer as broker might not be acting in the best interest of the client. Rule 1.7: Conflict of Interest: Current Clients (a) A lawyer SHALL NOT represent a client if it involves a concurrent conflict of interest When does that happen? There are two possibilities (1) OR (2) (a)(1)will be directly adverse to another client; OR Representing husband and wife in a divorce, etc. (a)(2) there is a significant risk that the representation of one client will be materially limited There's a significant risk that you're not gonna be the lawyer you otherwise would be to this client because of other responsibilities you have. PL: E.g., there is a risk that the broker-lawyer will not be sufficiently cautious in lawyerly advice not to do the deal that would yield the lawyer as broker the commission. It doesn't have to be certain only a significant risk (b) comes in when there is a concurrent conflict of interest under (a). The lawyer can represent the client anyway if four elements are met. Note the AND between (3) and (4). (1) competent and diligent representation to each affected client This is really the biggest thing b/c the attorney has to determine for themselves whether or not they can provide competent and diligent representation to each affected client. There are some conflicts that are nonconsentable, you can't even ask for consent. (2) the representation is not prohibited by law In GA you cannot represent two defendants in a capital case (3) representation does not involve assertion by on client against the other; AND (4) informed consent, confirmed in writing. 1.7(b) (2) and (3) are rarely in play but be aware of them With all that, can the attorney do this? There is a definite concurrent conflict of interest Is this consentable? Can you reasonably be competent and diligent to the client under these circumstances? If it is consentable and all three of the other requirements are met, including the informed consent, then the attorney can do what they need to do. Usually getting the informed consent is the problem if you deem the conflict to be consentable. My opponent's firm is an occasional client (pg. 220) The law firm representing your opponent is sometimes a client of your firm We represent a party against a party that is represented by a law firm which we represent from time to time. 24

Law of Lawyering Class Notes If I, as a member of a law firm, have a conflict of interest then all attorney's in my firm share my conflict of interest. 1.10: Imputation of Conflicts of Interest. Because we're associated in the same firm we share the 1.7 conflict of interest The actual problem is that we represent the other parties law firm from time to time. Why is this a problem? They are at least a former client PL: And significantly, a possible source of future business. The conflict here comes from the personal interest of the lawyer in not irritating the other law firm, a current client or a once and maybe future client. There is a basis for reasonable belief that they are our client Their firm has episodic dealings with our firm whenever they have issues that arise. Is there a conflict? Try boiling it down to what an independent attorney would in comparison to what the attorney in this situation would do Is the representation directly adverse? Are you going to be able to be the lawyer would otherwise would be if they were not your client? Is there a significant risk? Are you gonna be as willing to piss them off? Probably not, you're probably not gonna do what you would normally do if the firm wasn't our client. There's a concurrent conflict of interest because of your own personal interests. The conflict becomes evident in the midst of the representation, so what do you do? This conflict of interest arose during the representation and in fact in the midst of litigation Stop and thinkIs this Consentable? If you determine that this is not consentable, what do you then do? To continue with the representation would violate the rule so you can't do that. Under 1.16(a)(1), the representation will violate the rule. You do have to adhere to applicable law 1.16(c) If you determine that this is consentable, what do you do? You have to obtain informed consent from your client and from opposing parties firm (our other client). The consent has to be informed, remember what that means. The client says we messed up (pg. 235) Suppose first that in the midst of this complicated multimillion dollar deal you realize that one of the lawyers you had assigned messed up and it amounts to malpractice. You have to think about your malpractice insurance and you call your insurer. Tell your boss!! Do you tell your client? Are you obliged to tell your client? Rule 1.4(b) requires you to tell the client b/c the client now has a decision to make about whether to fire you or not, sue you or not. There is a duty to speak!! Once you've told the client do you then have a conflict of interest? 25

Law of Lawyering Class Notes Do you have a material limitation conflict that you need to at the very least discuss after you've told your client that you messed? We'll start with this next class. Part II of Nixon's Note-taking for Knuckleheads Who Never Note-take for Our Network of Near-Lawyers

Handout for Corporate Privileges Quiz


o

Rule 1.7(a)(2) issue

The attorneys must reasonably believe that they will be able to provide competent and diligent representation to each affected client

It must not be prohibited by law The attorney cannot represent both sides to a litigation Informed consent indicated by a writing would be required by both parties PL: note that the rule requires the informed consent to be confirmed in writing. This could be a letter to the client from the lawyer. It need not be a writing signed by the client, although that is a more prudent practice.

This means that full disclosure of the advantages and disadvantages to the representation must be disclosed along with the actual interests the conflicted attorneys may have

However, this is probably not a "consentable" representation based on the facts under 1.7(b)(1)

Husband and wife in same firm Husband v wife as counsel PL: This is the one raised in the question and discussed in class.

Husband party v wife party

Rule 1.8(e) issue

Only if they change the rules. But everyone does it anyways, unlike 1.8(j), which is hopefully rarely implicated. PL: Everyone would be an overstatement, but I am told it is common to violate 1.8(e). Rule 1.8(j) is a rule referred to but not discussed yet in class, and it is the rule that deals with sexual relationships between lawyers and clients.

Rule 1.8(a) issue PL:

26

Law of Lawyering Class Notes

Transaction must be fair and reasonable to the client, with all terms disclosed and understandable by the client

Client is advised about seeking independent counsel and is given the reasonable opportunity to seek independent counsel

Client gives informed consent, always in writing

But see Rule 1.5 Comment 4


Lawyer may accept property as payment for services Still must be reasonable as a fee under 1.5(a) Falls under 1.8(a) as well

The Client Says We Messed Up


o

Your own client may have a cause of action against you for legal malpractice

Duty - professional relationship Breach - failed the client somehow Causation - but for the malpractice, client would have gotten a better deal/would not have entered into the bad deal

Damages - value would have gotten/value of harm suffered

Irreversible error PL: I am not sure what irreversible error means in this context, but I do believe there is a duty to report your own malpractice.

Still must communicate with client

This puts yourself squarely in the crosshairs for a malpractice suit when you admit a breach of your duty to the client

Client wants you to continue Conflict of interest

Client's interests are directly adverse to yours at this point PL: Once you start the internal investigation for the sake of preparing to defeat the clients claim, I think you have entered the realm of direct adversity. We first discussed a material limitation type of conflict, one that arises because of the possibility that you could give advice that would lead the client to not go through the deal for reasons other than your malpractice. Such a course of action would eliminate the possibility of a malpractice suit (no damages) and thus there would be a significant risk that 27

Law of Lawyering Class Notes your representation would be materially limited, and the client needs to know about and, if possible, consent to this conflict.

You have to tell the client how exactly you messed up and what effect it has on the representation

Disclosing that information is lethal in a malpractice suit

Complex judgment is not a ground for malpractice suit

So, advise the client, based on your complex judgment, that the client perhaps should back out of the deal

This reflects a personal interest in avoiding the injury/damages of the malpractice

Thus, concurrent conflict under 1.7(a)(2)

Significant risk that the representation of the client would be materially limited by the personal interest of the lawyer

Self-awareness v self-discipline PL: I would say self awareness AND self discipline because you need to recognize the possibility that you would be tempted to give self-serving advice and you must be able to resist that. If you do not reasonably believe you can resist it, then you have a non-consentable conflict.

Consentable under (b)(1)?

Yet another complex judgment that must be made

Get to the bottom of the alleged error?


Could determine that the client was wrong Might as well prepare for the potential litigation Managing partner is responsible for things like this

Are they represented by counsel for the purpose of malpractice?

4.2 comment 7 determines whether you can even talk to them about this PL: Here we used the internal investigation scenario to illustrate the adversity and to review a little about 4.2.

Does an internal investigation create a conflict of interest?


Whole purpose for the investigation is to defeat a malpractice suit 1.7(a)(1) directly adverse to the interests of the client 28

Law of Lawyering Class Notes


Lawyer shall not represent the client unless (look to (b))

Call malpractice insurance carrier as first step

They will obtain a defense team for you and they will do whatever investigation is necessary to construct a defense

This probably doesnt put you in an adverse situation PL: By which I mean specifically direct adversity under 1.7(a)(1). Clearly in some sense you are adverse to your client, and you will need to inform your client that you have retained counsel to investigate, at which point it is likely that there will be a parting of the ways.

That they said malpractice enables the attorney to use otherwise confidential information in order to provide for the defense of the attorney under 1.6(b)(4)

Murder One, Murder Two


o

Both clients seek representation from the same counsel

If capital case, couldnt do it in GA

There are 35 comments to 1.7

Comment 24 refers you to 1.7(a)(2)


Codefendants or coplaintiffs Ordinarily, an attorney should refuse to represent codefendants in a criminal case because of the heightened potential for a conflict to arise

Detecting different degrees of culpability would create the potential for a directly adverse situation

Probably not a consentable situation PL: Think about this. Could you competently and diligently represent two criminal co-defendants with different degrees of culpability, which means that one may be able to make a deal if he rats out the other client? Surely you cannot, and therefore it is not consentable.

Most conflict analysis should be done at intake, especially with criminal defendants

One of the things that would help would be to study 1.18 on prospective clients PL: Which we will do later.

Represent both clients, one wanted to settle one did not, both end up being convicted for the same offense 29

Law of Lawyering Class Notes

Another lawyer comes in and claims ineffective assistance of counsel in violation of the Ds 6th Amendment rights

New trial

Strickland Test (hardest)


Did lawyer act in an unreasonable way in his representation? If not, then PL: this is not, under Strickland, IAC. If yes, then: D must show that there is a reasonable probability that, but for the counsel's unprofessional errors, the result of the proceeding would have been different

Holloway

Failure to investigate alleged conflicts require reversal without any need for prejudice

Cuyler (little bit easier than Strickland)

Did lawyer have actual conflict of interest that adversely affected the lawyer's performance?

Wheat

Presumption in favor of client's choice of counsel

Rebuttable by showing actual conflict or serious potential for conflict

Fire one client and keep the other


Former client issues arise Duties are owed to former clients PL: More on that to come Confidentiality issues

Can use information obtained during the course of the former client's representation that would be adverse to the former client

By "can" I mean it is possible, not permissible

1.9(c) Materially limited in the representation of the remaining client

Not really permissible, probably need to withdraw from both because of the increased risk of a conflict of interest

Was there a conflict? Yes Did it materially limit the representation or otherwise effect his performance? Yes 30

Law of Lawyering Class Notes

(Cuyler test)

"That which is not done." PL: This is the usual argument. The lawyer refrained from doing something that an unconflicted lawyer would have done, such a plea negotiations in which one client would have testified against the other.

When you perceive the conflict, alert the judge and ask permission to withdraw PL: This is the scenario where you are appointed to represent co-defendants. Once alerted to the conflict, the court risks automatic reversal under Holloway of the court does not at least investigate.

Conflict on pain of automatic reversal (Holloway)

Did Officer Schwarz Get the Effective Assistance of Counsel?


o

Worth represented codefendants Volpe and Schwarz

Volpe plead out and got a 30 year sentence

o o

1.8(f) if the PBA is paying Worth's fee Worried about the personal interests of the lawyer

Wants to have the $10mil contract renewed Best defense for Schwarz would harm the PBA and especially in its future defense of the civil suit charging conspiracy by the PBA PL: The actual harm is hard to pin down. It is certainly not formal, i.e. not res judicata or collateral estoppel. It could be a simple as the likelihood that using the wrong guy defense would be displeasing to the PBA, which was arguing the one guy defense in the civil suit.

Argument is that Worth chose his course of action based on the conflict

Worth recognized the conflict and explained it to Schwarz


Schwarz consented to the representation and waived the conflict BUT was it non-consentable?

Perhaps there was no right to ask for consent in this situation Thus, new trial

1.7(a)(2) significant risk of material limitation due to the lawyer's personal interest in the contract from PBA

Murder at the Ballgame


o

Prosecution seeks to disqualify the defense counsel based on a conflict 31

Law of Lawyering Class Notes


o

Remember, even if client gives informed consent, it does not necessarily mean that the representation was consentable

Wheat presumption

Court must recognize a presumption in favor of defendant's counsel of choice, but that presumption may be overcome not only by a demonstration of actual conflict but by a showing of a serious potential for conflict

Is there a limitation?

If yes, is it a material limitation? Look at 1.7(a)(2)

Need consent from not just Potero, but Pete as well

Longan says yes there is a risk of a limitation


You cannot present the defense of "it was Pete and not Potero" But it may not be material, since the representation may not require the fingerpointing to Pete

It is easier to disqualify PL: In some sense, it is easier for the judge to disqualify, because that decision will be reviewed under an abuse of discretion standard, but the court should do as instructed by Wheat and give some deference to the defendants choice of counsel.

But you end up saying to someone "you can't have the lawyer you want" This is why deference is given, and prosecution can make a motion

Party in the USA, Professionally. 1. Pop quiz class time party 2011. HYPO: We are in a meeting with Longan, whom has a business plan to start a business doing DIY divorce workshops. PL: Remember this exercise as an example of unbundling of legal services. a. Do it yourself divorce i. The way it works 1. 195$ workshop to represent yourself 2. Despite the concerns for the court, judges, and litigants nearly 50% of people who file for divorce do it without representation, so this could help many. ii. Problems 1. Is it reasonable under 1.2(c) a. Is it a reasonable? Yeah sort of. People do a terrible job on their own all the time (nearly half the time)

32

Law of Lawyering Class Notes b. Limit the scope of the representation to assistance with forms, and not a representative or advocate capacity to follow the divorce through the court system. 2. What is the practice of law? a. It is defined in the criminal code in the state of Georgia b. It is unlawful to practice law without a license. b. Completing the filing i. The way it works 1. Attorneys meet with both parties and help finalize the divorce 2. Simple completion of uncontested divorce PL: Think about the limits on the lawyers role, and the disclosures that would need to be made, in light of our later discussion of Theyre on the Same Page and What Kind of Consent? ii. Problems 1. Is it reasonable to ask for consent for the scope of representation to be limited simply to documenting the answers of the parties to an action? a. Its a tough one. Probably not. PL: This could work, as long as the lawyer is not advising anyone and is essentially acting as scrivener for parties who have come to an agreement. The problem is that the lawyer may recognize that one side is making an extremely bad deal in light of the underlying rights of the parties, and the lawyer acting as scrivener cannot alert that party to that problem. The parties need to be aware of the limitations of the lawyers role, including this one. I would suggest as a matter of prudence that you should avoid putting yourself in this position, but if you do you should get the consent in writing, with (among other disclosures) something like, Clients understand that lawyer will not alert or advise either Client regarding any potential rights or claims that either Client may have against the other Client, even if Lawyer is aware of such rights or claims. Clients further acknowledge that Lawyer has advised both parties that it would be more advisable and prudent for each Client to obtain an independent attorney to represent only that Client. c. Mediation i. The way it works 1. Allowed to be a neutral 3rd party ii. Problems 1. Under rule 2.4 and everyone needs to know that. d. Private legal consultations i. The way it works 1. Meet with you and discuss your legal jibjab about your soon to be ex-spouse. ii. Problems 1. Confidentiality issues, probably. No one said that though. PL: I am not sure what this means. The private legal consultation part of the business plan is the traditional model of full representation of only one party to the divorce. 2. Murder at the ball game pg. 252 a. This is, in part, nothing more than an application of 1.7. i. This would be a disloyal act to the client to wish to sell out 33

Law of Lawyering Class Notes ii. If you were using any information from him or about him, you will have to face the fact that the prosecution will put on your former client, pete and you will have to cross examine him. b. In part, there is a question of standing in WHEAT. i. Who has the right to gripe? 1. The prosecution, generally 2. There are reasons why we allow the prosecution to step in and interfere with the defendants ability to choose his own attorney. ii. Georgia rule 1.7 1. Primarily up to the lawyer to resolve conflicts 2. The court can question it. 3. where the conflict is enough to call into question the fair or efficient administration of justice, opposing counsel may properly raise the question. PL: Last comment to Georgia Rule 1.7 a. This is one of the limited circumstances that you would have the ability to complain that the opposing party should be disqualified based on a conflict. b. This is to prevent strategic abuse. c. Different standards on ineffective assistance. i. Hypo: represent pete. Dont use pete. Found guilty. Ineffective assistance claim. 1. Cuyler argument a. Had a conflict b. Affected performance ii. Hypo: doesnt represent pete. Dont use pete. Found guilty. Ineffective assistance claim. 1. Strickland argument a. A judgment call or strategy call by the lawyer, one with which a person could disagree, but the lawyer decides one way first prong. i. Enormous deference to the trial lawyer, but weigh both strategies and hypothetical outcomes. ii. We have to let the attorney make judgment calls and present the best defense. 3. Contributions to justice pg 263. a. Hitech company in the community wants the prosecution to go after people who sold trade secrets. Company offers to pay to prosecute. b. Preliminaries. i. Who is the client?: the state. ii. What is the goal?: prosecution of justice. iii. Discretion 1. There is a lot of wide discretion with prosecutors. 2. What should guide that discretion a. Equality b. Fairness c. Finances? i. Justice for the buck? How much can be done with the limited resources. 34

Law of Lawyering Class Notes d. Politics? i. Only if youre trying to get reelected. Then brandish the law as if it were a fiery sword of doom that guarantees another term in a lowpaying government job. PL: Politics should not guide discretion. See Nifong, Mike. c. Should the prosecutor accept the offer? i. Debate 1. Yes a. Im in charge. I have the discretion. I should do it. 2. No a. 1.8 informed consent for payment made by someone other than the client. Did the state consent? PL: I do not see this as a 1.8(f) problem. The lawyer is not being compensated for the representation. His office is being assisted with expenses. b. The prosecutor is tainting the process by accepting funds to prosecute. c. They are buying justice, which creates an obligation to pursue a prosecution that never would have been started. The results of the action will be driven by financial motives. ii. REAL ANSWER 1. California court said no. that it would create an expectation that is inappropriate. PL: The analysis is that the prosecutors discretion would be guided by an illegitimate set of factors, including a sense of obligation to a party who had invested so much money in a prosecution that perhaps the prosecutor would otherwise drop. iii. What if they provide the help the prosecution needs? 1. The class is split 50/50. I am the SCRIVENER so I am right! super totes good idea because it is just like a victim wins over super totes bad idea because its like taking the ducats. 4. Reasonable doubt. a. Prosecution has a good case. Case falls apart at trial. In her head, the prosecutor has a reasonable doubt. if I were on a jury, Id acquit. b. Preliminaries i. What do you do? 1. Make a deal. 2. Dismiss the case. 3. Probation. ii. What we are dealing with is a continuum. 1. Convinced I got the right guy -------> I have the wrong guy. 2. At some point, she should dismiss. At what point? c. ANSWER: given her state of mind, she should continue (gillers) 5. Will you represent us both 271. a. We are in a firm meeting. We vote together at intake. Clients are two persons who feel their employer discriminated against them. b. Preliminaries i. What can you get 35

Law of Lawyering Class Notes 1. Compensatory damages 2. Punitive damages and attorneys fees, dog. 3. Injunctive relieve 4. Procedural changes to promotional policies ii. Problems 1. Conflict under 1.7(a)(1). a. Only one of them can get the job b. So, one is right, and the other does not bolster the argument 2. 1.8(g) PL: This would come up if you proceed with joint representation and the defendant offers a lump sum settlement for all claims. a. In particular your clients need to know b. Everyone has to know what everyone else got. 3. 1.18 a. Duties with respect to prospective clients PL: More to come. 4. May say cant represent both; but, can we represent 1 or the other? 5. All we are going to seek is a declaratory judgment that discrimination is occurring and then seek equitable relief for whomever. a. Limiting the scope of employment under 1.2(c) b. As long as there is something that is available equally to both of them you can go forward. PL: Notice how the analysis of the conflict is driven by other law, particularly the law of remedies. You are looking for a remedy that would not create a conflict between the two prospective clients. 6. What kind of consent? - 271 a. 3 nice people who say they want to start a business doing birthday parties for children. Magician has the ducats, baker has the contacts, some other guy is doing another thing. They want to create a legal business structure. b. Problems i. The entity does not exist yet. So, the variances in legal interests in the speculative structure are substantial; and, each of their individual interests need to be considered as conflicts against one another until a structure agreement is reached. ii. Advocating for one direction could be adverse to one or many of the other clients. 1. If blood, sweat, and tears is considered equivocal value to financial equity, then the magician is entitled to no contribution on dissolution 2. If the money is considered debt then the magician is an insider creditor entitled to, upon dissolution of the entity, payment and satisfaction of his initial contribution prior to an division of the remaining assets to the other partners. 3. Failure to properly structure the businesses financial system (over-leveraging, under capitalization, etc.) would be bad for all of them together. iii. What can you do? 1. Counseling role. Advise them of the benefits and disadvantages of making decisions about the financial and management structures for their business. 2. Limit the scope of the employment a. I am here to advise you about the consequences of your actions, explain the law, provide insight as to future possibilities, but not to advocate one idea over another. PL: And presumably write it up. 36

Law of Lawyering Class Notes iv. Is there a conflict? 1. Yes. 2. Is it consentable? a. Yes. b. The limitation makes it consentable. v. 1.7(b)(4) content of the conversation 1. Communicate everything that is necessary for them to make the decision 2. Have to talk about the conflict and confidentiality 3. Explain to them why there is a conflict. There are things I just cant do because it helps one and hurts another. 4. Advantages 5. Risks comment 29 a. If it falls apart, additional costs, etc. b. You have to withdraw with respect to everyone. 6. Privilege wont attach; and, they could all be called to testify as to what was discussed in the process of entity creation. 7. The client can, and likely should, consider getting their own lawyers in order to be counseled on issues which the client for all three together cannot. 8. Comment 18: the effect on loyalty, look at comment 32. In particular here, you are seeking to establish a relationship. You say my role is not that of a partisan. (I have a limited scope) You are going to have to take the information I give you and you will have to decide for yourselves. 9. Comment 28: if you dont represent them all together, separate attorneys, greater costs, etc. PL: Note also Comment 31. 10. MIKE NIXON LIKES ROLLPLAYING WITH MAGICIANS AND PUPPETEERS 7. The insurer would like to know - 299

a. Is the insurance company a client? MAYBE. i. Duel client states say YES PL: Dual client model ii. 3rd party payor states say no. 37

Law of Lawyering Class Notes iii. Georgia is dumdumland, so no one knows. PL: The law in Georgia is unclear, although there is dicta that applies to this question in Georgia decisions. b. Is the insurance company a 3rd party payor? yes. 1.8(f) c. The attorney must give independent advice to the client. d. The problem i. Malpractice suit. Pom insurance, 2 defendants (law firm and partner in law firm), and lawyer trying to figure this out ii. at intake. 1. Should have a 1.7 conversation with both of the defendants. 2. No way to reasonably anticipate a conflict problem, but if you did, there would be no problem. 3. You are communicating under 1.4 with both of them. iii. Lawyer is reading emails and finds out that maybe Tipton sabotaged the deal because another competitor of malpractice plaintiff is a client. 1. This makes the situation intentional, not negligent 2. Not entitled to indemnification under the insurance policy. iv. INSURANCE COMPANY: CLIENT OR NO CLIENT 1. Yes a. By custom, attorneys do not report coverage issues to the insurer. b. They are required to, but they dont. c. Enter into an agreement under 1.2(c) to contractually limit the scope so as not to require disclosure of information to the insurer. PL Here is what the Restatement of Law Governing lawyers says on this subject: When there is a question whether a claim against an insured is within the coverage of the policy, a lawyer designated to defend the insured may not reveal adverse client confidential information of the insured to the insurer concerning that question without explicit informed consent of the insured. That follows whether or not the lawyer also represents the insurer as co-client. Note how this guidance differs from what a strict application of Rule 1.4 and 2.1 would require for a non-insurance situation. Not very satisfying of an answer, is it? The short conclusion is that the professional obligations of the insurance defense lawyer come from a complex web of rules that include not only rules of conduct but also contract law (the insurance contract) and custom and practice, as reflected in the Restatement. 2. No. a. 1.8(f) 3rd party payor. Law of Lawyering June 16, 2011 1) Review: Insurance, who is the client? 2) Restatement of the law governing lawyers (written out in more detail in last class notes) Pop Quiz: - Another client came to your firm from an insurance company. Your state follows the one-client model of insurance company representation. All your firms fees are being paid by the insurance company. You have just learned facts that, if true, would mean that your clients problem is not covered by his insurance policy, and therefore, the insurance company is paying your fee has no obligation to provide a defense of pay any damages that might be assessed against your client. Your firm relies on this 38

Law of Lawyering Class Notes insurance company for a significant number of cases each year. Can you disclose this information to the insurance company? Comments: PL: You have to look to 1.6 for authorization to reveal to the insurance company, which is not a client. The possibility we discussed in class was that the client might be engaged in insurance fraud, which would open the possibility under 1.6(b)(2) to reveal the coverage info to protect the insurance company. i) 1.6 (b)(2)- used your services to commit a crime or fraud. You have the option to reveal but it is not a mandate. Problem 2: The firm has been asked to represent three entrepreneurs who are putting together a new business. One is responsible for the money the new business will need to get started. One is brining essential technical expertise. The third has business contacts and sales experience without which the business will certainly fail. The entrepreneurs want the firm to represent all of them in the formation of the business. The conflicts of interest committee has been asked whether there would be any conflict in doing so, whether any such conflict would be consentable, and if so what must be discussed. ii) Comments: You have to limit the scope in order to make it consentable iii) Must inform them of the money situation, the pros and the cons of the limited scope. PL: See class notes from last time for detail. This is basically the What Kind of Consent? Problem from Gillers. Problem 3: A potential client informed a lawyer of the basic details and then the lawyer chose not to take him on as a client. PC found another attorney and the first lawyer now represents the other side. PC told attorney of many details. Comments: Did the client reasonably believe this lawyer was his lawyer? PL: If so, then there is an attorneyclient relationship. If not, then what obligations of loyalty and confidentiality still exist? That is what 1.18 is for. Has a rule been violated? 1.18- Prospective client problem 1.18(a)- Definition of prospective client- discussion with the lawyer the possibility of forming a client-lawyer relationship 1.18(c)- shall not represent a client with interests materially adverse to those of a prospective client in the same matter if the lawyer received information that is significantly harmful EXCEPT as provided in d. 1.18(d)(1)- both the affected client and the prospective client have given informed consent in writing 1.18(d)(2)- the lawyer who got the information took reasonable measures to avoid exposure to more disqualifying information than was reasonably necessary to determine whether to represent the prospective client AND that particular lawyer is out but the rest of the firm can do it. Screen out the lawyer. Defined in 1.0 You should ask who the parties are and who the lawyers are in order to prevent this. Has this lawyer used information other than what would be allowed under 1.9 (former clients)? This relationship is somewhat like the former client relationship. See rule 1.18(b). In this case, the lawyer should have stepped down and let someone else in his firm take over the case. This is a violation and a grievance. 39

Law of Lawyering Class Notes

Former Client Confidence PL: Conflicts 1. Divorce and Default (p. 309)- Conflict comes up in the midst of representation. (a) Is there a conflict? If there is a conflict, it should have been found. Conflict checks now do this often for law firms. (b) Look at 1.9 first. Patrick is a former client. Does he believe he is still a client? Is he an episodic client? Was there a letter sent by the attorney acknowledging the end of the client relationship? (c) 1.10- Imputation of Conflicts for rule 1.9- It doesnt matter which lawyer in the firm originally represented him because there is a conflict with all lawyers in a firm. (d) We want to protect former clients, but what are we trying to protect them from? Abuse of confidential information. PL: The main but not exclusive rationale for 1.9. (e) 1.9(a)- Cannot represent a client who is materially adverse to PL: The interests of a former client if the matter is the same or substantially related. So is this matter substantially related to the divorce representation AND materially adverse to the interests of the former client? (f) Wumcos interests is in being paid. This is directly adverse to the interests of Slip Shot. So it is adverse, the question becomes is it substantially related? (g) How do we figure out if it is substantially related? You can look at comments, will it involve any confidential information from the previous representation, etc. (h) Comments 3- What information would normally have been obtained during the course of the previous representation? The inquiry is what would you normally learn. 1. Would it materially advance the case against slipshot to know all of Patricks financial information? Yes. (i) You can end up with substantially related even if the cases dont looks anything alike. PL: And even if the parties are not identical (2) New Problem- done in class I. Lawyer 1 representing plaintiff versus Lawyer #2 representing Defendant doctor. Medical malpractice. The defendant wins. II. The plaintiff states he lost it because his lawyer committed legal malpractice. So it becomes plaintiff and lawyer #2 versus lawyer #1 III. Question: Does lawyer #2 have a conflict in representing her former adversary? a. Can lawyer #2 represent the plaintiff without giving away any confidential information against the doctor? He will have to prove that he would have won BUT FOR the malpractice of lawyer #1. b. The conflict is with Lawyer #2s former client the doctor. It doesnt matter that the lawyer was hired by the insurance company. c. Is #2 acting directly adverse to his former client? PL: No, but still a conflict with the interests of the former client It would be almost impossible for lawyer #2 to not use confidential information. d. The doctor would be the one who has to give informed consent, which wouldnt happen in this case.

Do I still owe the record store? (p. 310) i) It is substantially materially related. 40

Law of Lawyering Class Notes ii) Does this have anything to do with the confidential information? What is my prior representation? What would I have normally learned during my development of the lease? iii) Maybe the lawyer could be required to be a witness. Rule 3.7- Lawyer as Witness- concern behind this rule is jury confusion. General rule is that you cannot be a witness and an advocate. But you can be a witness when your partner is an advocate. iv) Rule 1.9 (former clients). The issues are the lease, whether there is excessive noise, and enforcement of the contract you helped negotiate. It is hard to imagine a circumstance in this situation where confidential information would be in play. There is more to this former client business than just the confidential information. There is an element of disloyalty. You ought not to be going against your client. PL: It is often said that this type of conflict involves attacking your own work, here the lease and the rezoning. It is a question of loyalty rather than the much more common former client problem, which is about the abuse of confidential information. nd o 2 problem- the answer is still the same, you are going after your own work. o 3rd problem- Competing record store. Can you represent an economic competitor? In this circumstance where it is materially adverse just as an economic matter. We look at it differently for policy reasons. PL: The policy being the encouragement of lawyers to specialize in representing particular types of enterprises, even if they are economic competitors. 1.7 (comment 22)- Advanced Waiver; Get prior permission from your client. Since this is in the future, it is harder for it to be informed consent. This might be ok for an advanced client. PL: This was a digression raised by a student question. Go back to Rule 1.9- What are the interests we are concerned with here? What would she normally learn during the representation of the business and the business plan? This could substantially advance the new record store owner. Do we look at economic interests differently than other issues? If we were to have this then we can look at 1.9(c). 1.7(comment 6)- simultaneous whose interests are only economically adverse are not a conflict and dont require informed consent from anyone; this is where you see the competing policy information. Economic adversity is not enough to prevent a lawyer from representing a competing benefit.

Hot Potato Problemo Your firm is representing a new client (client A) in a dispute with the local zoning board. The matter is in its early stages, and several other firms in the area could handle it as well as your firm. A larger existing client (Client B) has just called to ask your firm to represent it in a litigated matter against Client A. The case would be unrelated to the zoning matter. Although the new matter is not large, your partners very much want to please Client B by withdrawing from the representation of Client A and undertaking the new matter for the Client B. How do you advise your partners? You cannot sue your own client. 1.7 Can I withdraw from representing Client A to sue them for Client B? 1.16. 41

Law of Lawyering Class Notes 1.16(b)(1)- withdrawal can be accomplished without material adverse effect on the interests of the client. Can I do this under 1.9 (former client)? Has to be same or substantially related. Under Rule 1.9, can I sue a former client on an unrelated matter? Yes. But what will the court do about it? The Court can disqualify you under the inherent power of the court to control affairs. PL: This is just a rotten thing to do, and the court will prevent it if the matter is in court. See Picker. Hot potato p. 322 You dont know anything- pg. 329 o The migratory lawyer. A client of her new firm wants her to sue a client of her old firm. Lawyer has been with the new firm for 2 years. o Can the lawyer Sherry do this? Former Client- even though it is a different firm and 2 years ago. Is there a conflict? Rule 1.9 Not the same matter Substantially related matter- represented them against discrimination and now she is suing them for discrimination. o What would a lawyer normally learn during representation? Other instances of discrimination- how they are processed, settled, routine, practice, settlement amounts. Discriminatory attitudes HR policies Salary structure o Would it materially advance the cause of the new client? If all you know is there general knowledge of policies and practices, this is not enough. (Comment 3 to Rule 1.9). But knowledge of specific facts is enough to preclude. o Can her firm do this? Can you screen out of this? Yes, she has a conflict but should we impute it. Generally, this is a subject of great debate. Are we confident enough that if we set her aside that the previous client will be protected? 1.10(a)- Imputation of Conflicts- 1.10(a)(2)- This is not a general rule that says you can screen out of it. This is ONE specific area where you can be screened of it. There is a lot of procedural protections here. Can we hire Taylor Monk? o Small firm Taylor is coming from has a lawsuit against one of the large firms clients. o 1.9(c)- Not to use information for the detriment of your former client. Must go to 1.10 and do all that is required. However, the matter is still pending. o Not an inquiry about substantially related because it is the same transaction. It would simply be switching the sides. o When she leaves the firm does she just take with her personal conflicts or all the imputed conflicts?

42

Law of Lawyering Class Notes o If she has not acquired any confidential information, she does not have a conflict as she comes into our firm. But if she has confidential information, she is infected with the conflict. o We have a competing policy for the benefit of the public and for migrating attorneys. PL: That policy is to make it easier to migrate when client interests are not threatened. So when the lawyer has client confidential information and switches sides, then the lawyer has a conflict and migration is harder. If the lawyer has no confidential information, the clients interests are not threatened, the lawyer has no conflict, and migration is easier. What if a few years later Taylor leaves the firm and takes Admiral with them? Then a new client comes to the firm and wants to sue Admiral. What circumstances, if any, can the firm take on the new client? LOLing it up 1. Pop quiz questions a. Migratory attorney i. firm considering hiring Gretchen Mitchell, whom spent last several years representing the manx corporation against feral incorporated. Can new firm hire her if the firm represents feral incorporated. ii. Issues 1. We want to keep the case. 2. We want to hire her iii. Answer 1. Is there a conflict? Yes. a. Migratory lawyer b. Carries her duties to former clients with the transfer c. Ask if she has information which is protected under 1.6 or 1.9. i. She will. She represents the a/p. ii. So she has a conflict. d. 1.10: screening i. If she can be properly screened in conformity with 1.10 then she can be hired. ii. Screen, notice, etc. (make sure to indicate that you know the application of 1.10 to issues of conflict) b. Terminated corporate company i. Firm formerly represented Charlie and Emery, inc, a corporation that owns and operates a chain of fitness centers. Scope limited to creating a manual for purposes of compliance with anti-discrimination laws. Services terminated 6 months ago. Can you represent a plaintiff against the company for discrimination? ii. Answer 1. Substantially related is defined by comment 3 to rule 1.10. 2. If it is substantially related, there is a conflict which prevents representation. 3. Depends on if all you know is policies and procedures under comment 3 to rule 1.10. 4. If it is, you are good. If it is not, then you are not (PL: For example, you learn specific facts that would be helpful in the second representation, such as the boss is a bigot, specific facts that would conflict out the representation.) 43

Law of Lawyering Class Notes c. Denied prospective client i. Firm declines to represent prospective client after consultation where prospective client blurted out more than necessary after being told just tell me just enough. Firm wants to represent the other side. Can they? ii. Answer 1. Under PL: This is actually under 1.18 1.9 the firm can undertake the representation if reasonable efforts were taken. 2. May have been. 2. The firm left behind a. Hypo: taylor monk joins a firm. Represents admirable incorporated in a defense against a class action. Taylor monk tries to get a raise, doesnt, leaves, takes the client and the case with her. The firm is left behind (witty). Class in that action wants to replace lawyers and hire taylor monks old firm. Can they? b. Issues i. Migratory attorney problem, only what happens to the firm when the attorney leaves. ii. Conflict issues iii. Duties to former clients c. Answer: i. 1.10 (b)(1) 1. This is a lawyer terminating an association with a firm. 2. The firm is not prohibited from thereafter representing a person with interests materially adverse to those of a client represented by the formerly associated lawyer and not currently represented by the firm UNLESS a. The matter is the same or substantially related to (it is the same) AND b. Any lawyer remaining in the firm has information protected rules 1.6 and 1.9(c). i. SO, if all attorneys who worked with taylor on the case leave with taylor, or leave generally, then youre good. ii. What if there is an attorney with 1.6 or 1.9(c) information? 1. 1.10(a)(1) a. Doesnt help you b. The facts do not indicate this is a personal interest of an attorney 2. 1.10(a)(2) a. Screening doesnt help you b. Screening helps the hiring of an attorney with a conflict, it does not cure a conflict for an already associated attorney. iii. NOTE 1. What about non-attorneys with access to or actual knowledge of 1.6 or 1.9 information? a. Rules of professional conduct only apply to lawyers b. 5.3 i. A Lawyer has the duty to make sure that the people that are not lawyers that work for the lawyer do not do things that the lawyer himself/herself couldnt do. ii. Make sure your support staff isnt a bunch of blabbermouths. 44

Law of Lawyering Class Notes c. You kind of have to screen your staff. d. 1.10 comment 4 i. Talks about law students, paralegals, secretaries, etc. ii. Actually uses the language screening. 3. Investigating landlords variations: Generally, problems on movement from government service to private practice. a. Question 1 i. Chen sued slumlord as a governmental employee for activities at the green, a complex of apartment buildings. She obtained a cease and desist order. Chen leaves government, and her new firm is asked to represent the slumlord in defending against a contempt citation for violating the cease and desist order. Is chen disqualified from participating personally in the defense of slumlord. ii. Answer 1. 1.11 a. Chen will be subject to 1.9(c). (1.11(a)(1)) b. Cannot represent a client in connection with a matter in which the lawyer participated personally and substantially as a public officer or employee unless the government gives informed consent. (1.11(a)(2)) c. If disqualified, no one PL: in the firm can represent the client which creates the conflict without first screening (1.11(b)). b. Question 2 i. Chen sued slumlord as a governmental employee for activities at the green a complex of apartment buildings. She obtained a cease and desist order. Chen leaves government, and her new firm is asked to represent tenant in seeking damages for the violation by slumlord that led to the cease and desist order. ii. Answer 1. This time shes on the same side as the government. 2. 1.11(a) except as law may otherwise expressly permit, a lawyer who has formerly served as a public officer or employee of the government: 3. Shall not otherwise represent a client in connection with a matter in which the lawyer participated personally and substantially as a public officer or employee. 4. 1.11 comment 3 calls this an exploitation of public office. c. Question 3 i. Chen sued slumlord as a governmental employee for activities at the green a complex of apartment buildings. She obtained a cease and desist order. Chen leaves government, and she got to see the tax returns. ii. Answer 1. 1.11(c) a. the lawyer has now confidential government information. b. Cannot represent anyone adverse to the slumlord where that information could be used to a material disadvantage. d. Question 4 i. What about the firm? ii. Answer 45

Law of Lawyering Class Notes 1. 1.11(c) a firm with which the lawyer is associated may undertake or continue the representation; but, the attorney must be screened. e. Question 5 i. Suppose chen represented the slumlord in the cease and desist action. Chen goes to work for the government (for whatever reason that would compel a person to do so) can she now represent the government against landlord. ii. Answer 1. 1.11(d): no. f. Question 6. i. If chen is disqualified under rule 1.9 is her entire department at the government now disqualified. ii. Answer 1. Firm disqualified, get another firm. Government disqualified, get a new government. PL: This is a way of articulating the choices to understand why the lawyers conflict is not generally imputed within the government agency. 2. 1.11 comment 2: because of the special problems raised by imputation within government agency, paragraph (d) does not impute the conflicts of a lawyer currently servicing as an officer or employee of the government to other associated government officers or employees, although ordinarily it will be prudent to screen such lawyer. 4. Quiz 2 a. Client is the local public defender. Pd employs 3 apds. Pd office appointed to represent 3 ds, all of whom are accused of participating in an attempted armed robber. Client wants advice regarding the possibility of having one apd represent all 3 ds and the possibility of having each of his 3 apds represent one of the ds. Advise your client regarding these matters. b. Answer: i. What does a firm mean? 1.10 comment 1. The answer is not clear. 1. The class is split 50/50 on this. 2. If it is a firm there is a conflict under 1.9 that cannot be imputed under 1.10. ii. Answered in an ethics committee advisory opinion. Said it was a conflict. 1. Answer being contested by Supreme Court of Georgia 2. Longan thinks it will be overturned. PL: That the Supreme Court will reject the opinion that said that conflicts within a circuit defenders office ought to be imputed. Well see. 5. Fidelity to Court a. Nix v. Whiteside and dozens of adorable variations thereof. i. Facts: someone comes into the bedroom of another. There is a confrontation. One kills the other. Self defense is the issue. Whether or not the defendant (surviving party) had a reasonable fear of imminent harm. Did our defendant see the other party with a gun? He at least saw the defendant reach under the pillow. There was some reference to the victim saying get my piece (not, reeses, I presume). ii. NOTE 1. knowlingly definition 2. 1.0(f): knowing, knew, circumstances infer that you did know. 3. 3.3 comment 1. This mess totes applies in a deposition. 46

Law of Lawyering Class Notes iii. Variation 1: PL: Civil case 1. Whiteside stabs love and kills him. Loves estate sues whiteside for wrongful death. Whitieseide tells you that he is going to testify that love had a gun, even though whiteside admits to you that this is a lie. What do you do in order to make sure you comply with the model rules of professional conduct. 2. Answer a. Talk to whiteside. b. 3.3(a)(1)(3): a lawyer shall not knowingly offer evidence that the lawyer knows is false. c. I cant do that, brother whiteside. i. 1.16(a)(1): withdrawal because you cannot continue the course of litigation without violating the MPRE ii. tell the judge cant continue to represent my client and without violating the rules of professional conduct 1.16(a)(1). Judge says tell me why iii. 1.6(b)(6) cant tell you without being compelled by a court order. Compel if you so desire, judgy. d. 3.3 Comment 6 iv. Variation 2 1. Whiteside tells you that he saw love with a gun before he stab bed love. You dont believe whiteside, but you are not sure that he is lying. Whitside insists on testifying that he saw the gun. What are your options. 2. Answer a. 3.3(a)(3) a lawyer may refuse to offer evidence, other than the testimony of a defendant in a criminal matter, that the lawyer reasonably believes is false. b. Objectives and means i. Objectives belong to the client ii. PL: General rule about means Means belong to the lawyer AND client 1. Consultation on means 2. May or may not agree. If no agreement, can withdrawal or something. c. 3.3 comment 9 PL: 3.3(a)(3) and comment 9 may be seen as a special rule about one particular means, the use of evidence that the lawyer reasonably believes is false. That decision belongs to the lawyer. v. Variation 3 1. What if he gets up on the stand and says I saw a gun. You didnt know he was going to say that; but, you know its false 2. Answer a. 3.3 comment 10: in such situations the lawyer must take reasonable remedial measures. i. In such situations, the advocates proper course is to remonstrate with the client (take him by the scruff of the neck and say: this aggression will not stand, man. J. Lebowski). 47

Law of Lawyering Class Notes ii. Advise the client of the lawyers duty of candor to the tribunal and seek the clients cooperation with respect to the withdrawal or correct of the false statements. b. If that fails, the advocate must take further remedial action. If withdrawal from the representation is not permitted or will not undo the effect of the false evidence i. This is a must circumstance ii. Mandated in order to maintain the lawyers duty to protect the system. vi. Variation 4 1. Criminal case. Whiteside is on trial for murder in the stabbing of love. He tells you that he intends to testify that love had a gun, which you know is false. 2. Answer a. Under 1.6(a) PL: This is actually 1.2(a) the decision to testify is usually a shared decision between lawyer and client because it is a means. b. 1.2 in a criminal case the attorney will abide by the clients decision to testify. c. 3.3 comment 7 i. Duty PL: not to present false evidence applies to all lawyers. ii. Some jurisdictions make you let your lying defendant testify in the form of narrative without the assistance of counsel. When that happens, those rules subordinate the obligations under the MRPC. d. You hope that you will not be held professionally responsible for violating the rules of professional conduct at the order of the court. PL: You would not be. See the language in comment [7] The duties stated in paragraphs (a) and (b) apply to all lawyers, including defense counsel in criminal cases. In some jurisdictions, however, courts have required counsel to present the accused as a witness or to give a narrative statement if the accused so desires, even if counsel knows that the testimony or statement will be false. The obligation of the advocate under the Rules of Professional Conduct is subordinate to such requirements. (emphasis added) vii. Variation 5 1. Criminal case. Whiteside is on trial for murder in the stabbing of love. He testifies he saw a gun and you know this is false. 2. Answer: same as civil viii. Variation 6 1. Criminal case. Whiteside is on trial for murder in the stabbing of love. He wants to testify he saw a gun, you think it might be PL: false. 2. Have to let him do it. 3. 3.3 comment 9 a. Reasonably believes but does not know? Have to let the liar talk.

48

Law of Lawyering Class Notes b. UNLESS THE LAWYER KNOWS THE TESTIMONY WILL BE FALSE ix. Variation 7 1. I did not actually see a gun, but I am convinced love had one. I saw something metallic in his hand. If I dont say I saw a gun, Im dead. 2. Answer a. When you know things is tough. b. If I dont know the obligations go away. c. I think therefore I am. d. THIS lawyer decided this would be false. b. Out carousing with mikey 376 i. Criminal defense lawyer. Has a client carlo. Defense is alibi. Carlo was out carousing with mikey. Mikey is a BFG PL: For those of you who do not remember your Roald Dahl, this means Big Friendly Giant. . Remembers that he was out drinking beer with carlo. Lawyer puts carlo on the stand. I was out at the dock boozing it up with mikey. Mikey takes the stand I was boozing it up with carlo. After the fact, mikey tells lawyer I just realized that it couldnt have been me. That was in august. I know that because it was my daughters PL: Nieces birthday and I went to visit her. Here is a bunch of evidence that proves I wasnt with carlo. PL: bus ticket, ATM receipt, I-pod receipt ii. Answer 1. 3.3(a)(3): offered evidence and have come to know that it is false. 2. Must take remedial measures c. Lawyers lecture i. Dialogue with client who is digging for an excuse for murder. Is it wrong for you to lay out the law prior to the story so that the client is inferentially encouraged to conform the story so that he has a viable legal defense? ii. Answer 1. This becomes a matter of lawyers conscience. This is something that says you cant assist them to testify falsely. PL: This is largely a matter of conscience because the assistance to a witness occurs behind closed doors, under the apparent protection of the privilege. Who will ever know? It is a difficult issue even for the lawyer who wants to comply with the rules because the lecture certainly can be a clever way of coaching a witness into a false story, but (as several of you pointed out) there are legitimate reasons for the lecture as well. 2. What falsely means is hairy; so, be careful when walking this line. d. In the Matter of Bill Clinton i. A bunch very specifically worded sexually related questions. ii. Some are blurring a lot of lines and there arent any quick, easy answers. Its important to parse the words appropriately and avoid over-exposure of your client to liability while still maintaining your obligations under the MRPC. iii. The most shocking is the one where the lawyer misspoke and said Martha instead of monica. Bill answers honestly the questions; but, the lawyer meant something else; and, what the attorney meant is somehow inferred from the context. 1. dezarn case. Called a similar situation perjury. 2. Lesson for us: know what perjury is. 49

Law of Lawyering Class Notes PL: Remind me and I will post the exact questions and answers to the course page. The difficulty is in deciding when it is the other lawyers job to ask better questions and when you have the obligation to do something. The hardest, and most surprising example, is the one in which the questioner obviously misspoke and said Martha instead of Monica. See DeZarn. e. Did you communicate with cassie. 410 i. What did this witness know about the financial situation of the bamoboozle corporation. What the attorney wants to prove is Cassie communicated this crucial information to the witness, and the witness therefore commits fraud. Witness says I did not communicate with cassie. she sent me a voicemail said later to the lawyer. I didnt communicate with her, she communicated with me. PL: After learning about Dezarn, the class seemed to come to a consensus that the testimony in this problem that he did not communicate with Cassie was literally true but nevertheless perjurious. I agree, although it is a harder case that the Clinton example or Dezarn. One important lesson is to make sure that you are not overly literal in your parsing of the questions and answers. Right or wrong, Dezarn tells us that a literally true answer can be perjury.

1. Pop quizathon 2011 a. Question 1 i. You have a substantial bankruptcy practice. Your client is the scrooge national bank. All of your bankruptcy lawyers left the firm and went to work for another large firm. They took all of SNBs matters and all of the attorneys who worked on the case. Left behind a law student who worked closely on SNB matters. You have been approached to represent a client against SNB in closely related matters. Under what circumstances, if any, may your firm undertake this representation? ii. Answer: 1. 1.10(b): No conflict. All attorneys left 2. Screen the law student. PL: Note comment 4 to 1.10 b. Question 2 i. You are representing Marie de Houston, a video game designer. You are suing MEL enterprises for copyright infringement. In the middle of trial, you learn that one of your clients fans has threatened one of your jurors with death if marie does not win and that the juror has been too afraid to tell the judge. Marie had nothing to do with this threat. What should you do? ii. Answer 1. 3.3(b): a lawyer who represents a client in an adjudicative proceeding who knows that a person intends to engage, (or will or has) in criminal or fraudulent conduct related to the proceeding shall take reasonable remedial measures, including, if necessary, disclosure to the tribunal. a. Note: i. ANY PERSON ii. Very tense: is, was, will PL: Every tense iii. Crime iv. Related to the proceeding v. We have seen reasonable remedial measures in 3.3(a) vi. There is NOTHING you can do but reveal it to the court. 50

Law of Lawyering Class Notes b. This includes i. Jury tampering ii. Threats on jury or prosecution or judge 2. Movie time a. 3.3 is a time when mays of 1.6 become must s i. False evidence ii. Must take reasonable remedial measures b. 4.1(b) is another 3. Duties to the tribunal ctd. a. Eyewitness part 1. 420 i. Defense attorney representing a guilty client. Client robbed bank. Confessed. Confession suppressed. Attorney trying to find a way to argue reasonable doubt. Witness saw defendant take off ski mask and run away. Lawyer wants to impeach witness. Lawyer wants to use a guilty plea for lying on a loan application. Defense lawyer has an alibi witness that puts the defendant at the burger king. ii. Answers 1. You CAN do this. 2. Should you? a. We all think you should. b. Longan proposes we are being deceitful PL: For the sake of argument. Nothing personal. 3. What about a prosecutor a. A prosecutor cannot impeach a witness the prosecutor knows to be truthful (*3.4(e)). PL: I am not sure where the reference to 3.4 comes in. 4. Reliable sources say a. There is an allegation of price fixing by the in house counsels client. There is a document called preservation policy whereby after a certain amount of time electronic documents are purged. What should the attorney say if b. 3.4(a) i. A lawyer shall not: 1. Unlawfully obstruct another partys access to evidence or unlawfully alter destroy or conceal a document or other material having potential evidentiary value. A lawyer shall not counsel or assist another person t do any such act; 2. Note a. You can do so if it is not UNLAWFUL b. What makes it unlawful c. The rules point you to other law here. i. 18 USC 1512 (title 18 is the crime title) ii. Hoever knowling corruptly persuades another person or attempts to do so, or engages in misleading conduct. Altering stuff something like that. iii. (c) corruptly alters, conceals, destroys a document with the intent to impair the objects integrity or availability for use in an official proceeding. d. Official proceeding? 51

Law of Lawyering Class Notes i. Proceeding need not yet be initiated. ii. Expected in the foreseeable future is enough. e. The point is i. There is no definitive answer ii. There is irreducible uncertainty. 5. Ryder/Meredith a. In re ryder: version #1 i. I just robbed a bank. What should I do with the gun? May you advise your client to destroy it? ii. Answer 1. No. 3.4(a): counseling someone to unlawfully destroy. b. In re ryder: version #2 i. The gun and the money are in my safe deposit box. Go get them and keep them until this blows over. Can you do this? ii. Answer 1. No. 3.4(a): concealing c. In re ryder: version #3 i. Ive been arrested for robbing the bank. There is a file fo.lder on my desk in my house labeled bank robbery Plans. Go get the file and hold it until this blows over. Can you do this? ii. Answer 1. No. 3.4(a): unlawfully concealing. PL: This time it is evidence, rather than the fruits or instrumentalities of crime. Answer is the same. d. In re ryder version #4 i. You take possession of the plans. The prosecution subpoenas them from you. May you resist production on the basis of the attorney client privilege? ii. Answer PL: The document will be privileged in your hands only if it would have been privileged in the clients hands under some theory, such as self-incrimination. 1. Not privileged. This is not a communication. 2. This is subject to subpoena. 3. 3.4(a): bad news bears. PL: ? e. In re ryder version #5 i. Ive been arrested for a robbery with a sawed-off shotun. I have sucha a gun, but I did not commit this crime. My gun is in my safe deposit box. Go get it and have an expert test it to see if it has been recently fired. May you follow your clients instructions? ii. Answer 1. 3.4 comment 2: you can do it without destroying it. PL: Then you must turn it over tp the police or prosecution. 6. People v. Meredith variations a. Generally, victims wallet in a burn barrel, charred but not destroyed, behind the defendants house. b. Meredith variation 1 i. Your client tells you that the wallet is behind his house. You do nothing. Can you be compelled to disclose the location of the wallet? Are you guilty of obstructing justice. ii. Answer 52

Law of Lawyering Class Notes 1. Not obligated to disclose. This is a privileged communication. 2. Fairness. Not my job to do their job. Meredith variation 2 i. Your client tells you where the wallet is. You go observe its location but do not disturb it. Can you be compelled to give its location? ii. Answer 1. This is an observation that flows directly from the communication. 2. So, this observation is also protected by the privilege. Meredith: version #3 i. Your client tells you where the wallet is. You find it and take possession. You turn it over to the police. Can you be compelled to testify or stipulate about its location? PL: Yes. ii. Answer 1. Made it impossible to locate where it is. Its location is its significance. In a certain sense, the evidence is altered. iii. Sub-variation. Take it back to the office. Examine it. And then take it back and put it back where it belongs. PL: Problem is they may have looked for it there in the meantime. Meredith version #4 i. Your client personally hands you the wallet. May you be compelled to testify that your client gave it to you? If so, what do you do when the client tenders evidence to you? ii. Answer 1. There is no definitive answer 2. Some cases say you have to say where it came from 3. Some cases say you dont have to 4. WHAT TO DO a. Unless there is a reason to take it, DONT b. Tell the client I cannot counsel you to alter or destroy that. It would be obstruction and I will not engage in that. i. Under 1.2(d) you can talk to your client about it but you cannot assist them in commiting any crime. Striking a balance i. You can go look at it, whenever you want, so long as you dont move it because that is fair to the defense. ii. BUT, once youve moved it, youve altered it to a degree; and, it has created an unfair circumstance for the prosecution.

c.

d.

e.

f.

Transactional setting The bad builders good lawyer 504 o Suing a lawyer Building developed the building in which he promised a particular plumbing Circumstances made it problematic to continue without r She quit Pipes burst o Is there a base to complain of the lawyers conduct? 53

Law of Lawyering Class Notes o PL: first variation: My client put nothing but he best plumbing in this house. Its the top of the line Rule 4.1- truthfulness in statements to others; in the course of representing the client; making a misrepresentation of law o Is it misconduct if youre not representing a client? Rule 8.4(c)- it does not say in the course of representing a client This comes into play in business affairs of the lawyer There are states where there has to be some relationship to the practice of law, but the MRPC say it is wrong PL: Under MRPC, deceitful conduct unrelated to practice can still be professional misconduct. o PL: Second variation: My client ordered the best plumbing supplies that money can buy to put into the house for you. Its a misrepresentation; we want people to be able to trust lawyers and we dont want everyone to feel like they have ask a million questions PL: It is misleading, even if literally true; violation of 4.1 4.1, Comment 1 o The buyer volunteers and says I know you put the good stuff in here, but I dont want to pay that much. The lawyer hears her say that. PL: Hard question: when does the lawyer incorporate or affirm a statement of another person that the lawyer knows is false is a nod of the head enough? See Comment 1 to 4.1 o PL: Next variation: Lucy you should buy this house. It is worth $1.5 million dollars in todays market. Is it a misrepresentation of a fact? Everybody knows. Its a part of our culture. Puffing. 4.1, Comment 2 Why dont just let lawyers lie? Transactional costs We want people to trust what lawyers say is true. PL: It is efficient to have a rule that requires truthfulness, and simply permitting deception would be (at least to me) a demoralizing way to live, with everyone expecting you and everyone else to lie o PL: The actual problem: She didnt know anything before, but now, Lucy finds out that her client put ordinary parts into the house before the sale closed. She did not disclose that fact to the buyers, who bought the house and now must move out. Did she have the duty or option (or neither) to disclose under the MRCP? You would want to have a conversation. What does that sound like? You know this is fraud right? You tell them or Im gonna tell them PL: Note that this could be the conversation because you have the option under 1.6(b)(2) to reveal to prevent the fraud. On the other hand, you might not want to be, or be known as, the lawyer who ratted out a client. Do you really want to be the lawyer with the reputation of ratting out clients 1.2(d) sais that you know PL: The relevance of 1.2(d) is that you cannot assist in a fraud, and in this scenario your assistance could survive the termination of the attorney client relationship. 54

Law of Lawyering Class Notes You dont have any choice but to withdraw if they refuse to reveal If she quietly falls away PL: The assistance is still there. Either you have to make disclosure unnecessary (if the noisy withdrawal works) or you have to reveal under 4.1(b). There is something called a noisy withdraw 4.1 2.4 look at comments; noisy withdraws PL: Comment 3 to 4.1 and comment 10 to 1.2 4.1(b)- you have to speak Noisy withdraw might make it unnecessary o ie. The buyer runs away unless its prohibited by 1.6 PL: Remember this was language added on the floor of the House of Delegates of the ABA to gut this duty to speak, at a time when 1.6 was so much more restrictive that in this scenario there would not have been an option under 1.6 to reveal; now, however, the broader provisions of 1.6(b)(2) would not prohibit revelation of the fraud, and therefore 4.1(b) mandates disclosure if doing so is necessary to avoid assisting in a fraud. o 1.6 was changed and added 1.6(b) to allow for some things to be Its a violation, but there is no cause of action PL: The Rules do not create causes of action but could be evidence of a duty. The hard part with civil liability here is that the plaintiff would not be the lawyers former client. Lawyers have some limited duties to third parties, and the argument here would have to be that the lawyer owed a legal duty to speak or that, by not speaking, the lawyer was part of the fraud. Notes: Class 8 July 5th, 2011 Pop Quiz Questions o Partner told attorney to tell opposing counsel that under no circumstances would the settlement offer be acceptable. Attorney knows for a fact that this is a lie. Can he lie to opposing counsel? Rule 4.2 Comment 2 You have to be truthful in statements to others, but this statement would likely be a statement that would not be treated as a statement of fact by opposing counsel. If it was a closer call, remember Rule 5.2 where a subordinate lawyer can rely on a superior lawyers reasonable resolution of an arguable question of professional duty o Attorney figures out that client is using attorneys work to perpetrate fraud on buyers of her business. Partner says that they are fine as long as they did not know about the fraud when they did the work and that it would breach confidentiality to reveal the fraud. Is the partner right?

55

Law of Lawyering Class Notes No, according to 1.6(b)(2) or (3), you can reveal confidential information to prevent crime or fraud and Rule 4.1(b) (and Comment 3 thereto) creates an affirmative duty to that effect. You must withdraw representation and do so noisily by giving notice to parties at harm that you are no longer representing the client, though you do not have to tell them why you withdrew. PL: You also then may have the duty to reveal the fraud under 4.1(b) if doing so is necessary to avoid assisting in the fraud. The Case of the Complex Formula (pg. 506) o Version 1 Lawyers negotiate vigorously over the inclusion in the property settlement of a forum selection clause that requires any litigation over the agreement to be in Chesters wifes new state. Chester eventually agrees. Wifes lawyer accidentally leaves out that provision. May Chesters lawyer remind her? o Ask your client if you should take advantage of the scriveners error. PL: And give advice in this conversation, you are complying with your 1.4 duties and your 2.1 duties. There are many factors to consider here (and give candid advice on): Merger Clause (and whether the document will hold up without the forum selection clause even though there is an oral contract and written evidence to that effect) Financial Matters (if the forum selection clause is included, it will reduce or eliminate some later litigation on deciding the forum and save your client some money) Morality and Honor (discuss with your client the moral implications and how it is the honorable thing to do to stick 56

Law of Lawyering Class Notes to your word and include the clause) PL: remember, under 2.1 you may have this conversation but are not required to do so. o ABA says you have to tell the wifes lawyer about the missing clause but you do not have to consult with your own client. You should probably consult with your client regardless. PL: I do not think there is any doubt. Regardless of what the ABA opinion says, I think 1.4 requires a conversation with the client here. o Version 2 Wifes lawyer omits from the property settlement any consideration of the money Chester inherited during the marriage, apparently in the mistaken belief that the law does not permit her to claim any portion of it. Should Chesters lawyer tell Wifes lawyer about the mistake? o There is no duty to tell the other lawyer. Their lawyer messed up, and their lawyer is liable. o However, you should probably consult your client about this and ask them what to do. PL: Again, I would say this duty is clear. I would delete probably in this sentence. But also be careful. It is not that you are asking them what to do. You are consulting with them. This decision does not belong to your client alone or you alone. Do not assume your clients objectives. They may not be purely financial. o Problem as Written Wifes lawyer has made an arithmetical error in the drafting of an agreement that has not been agreed to. May Chesters lawyer agree to the bottom line and draft a simple

57

Law of Lawyering Class Notes agreement incorporating that term? Does he have the option to tell Wifes lawyer about the mistake? Agreeing to the bottom line when the itemized list is in the contract seems unconscionable. o Consult with your client about this matter. Factors include the obvious future litigation about the matter (the contract is likely to be reformed to reflect the intention of the parties) and the fact that it is clearly just an arithmetical mistake. You cannot tell without the clients consent. o There is no duty to speak since this is not fraud. PL: Here is an excerpt from the opinion in the actual case, in which the wifes lawyer had subtracted $308,362.99 from $550,000 and got $140,637.01: The mistake might never have come to light had not Tim desired to have that exquisite last word. A few days after Joan had obtained the divorce he mailed her a copy of the offer which contained the errant computation. On top of the page he wrote with evident satisfaction: "PLEASE NOTE $ 100,000.00 MISTAKE IN YOUR FIGURES. . . ." The present action was filed exactly one month later.

o Transactions with Unrepresented Persons: The Case of the Pro Se Divorce (you still represent Chester) Chesters wife does not have a lawyer. You contact her about the property settlement, and she says, thank you for helping us out. You should respond with I am not your lawyer and explain to Chesters wife your representation and your role in the matter. See Rule 4.3 (the lawyer shall make reasonable efforts to correct the misunderstanding) and Comment 1 thereto. 58

Law of Lawyering Class Notes You show her the property settlement and she asks is this everything I am entitled to? Again, explain to her that you are not her lawyer and you cannot give her legal advice and that she should seek legal advice on the matter. See Rule 4.3 (The lawyer shall not give legal advice to an unrepresented person other than the advice to secure counsel). Chesters wife asks, What does forum selection mean? See Comment 2 to Rule 4.3 where the prohibition to give legal advice does not apply since in this case it is necessary to even make the agreement in the first place. (So long as the lawyer has explained that the lawyer represents an adverse party and is not representing the person, the lawyer mayexplain the lawyers own view of the meaning of the document or the lawyers view of the underlying legal obligations.) Murphy & Demory, Ltd. v. Admiral Daniel J. Murphy (pg. 561 we read about it in Legal Profession) PL: I just thought you would find this interesting. This is not required reading for this course. Contraindications (pg. 564) o Who is Worzeks client? He works in-house at Lifecare, and this entity is his client. Rule 1.13(a) A lawyer employed or retained by an organization represents the organization acting through its duly authorized constituents. o What should Worzek have done if one of the researchers said, Theres something you should know, but it will have to be between us. I falsified some of the Amulex data. Rule 1.13(f) In dealing with an organizations directors, officers, employees, members, shareholders or other constituents, a lawyer shall explain the identity of the client when the lawyer knows or reasonable should know that the organizations interests are adverse to those of the constituents with whom the lawyer is dealing. o I looked at the paperwork we gave the FDA. It implies greater thoroughness than I was finding and none of the reservations. Is this confidential information under 1.6?

59

Law of Lawyering Class Notes Yes, since you learned these implications in the course of the investigation of the documents. Does Worzak have the option to reveal the information to the public under 1.6? Potentially 1.6(b)(1) however death or harm is probably not reasonably certain Potentially 1.6(b)(2) to prevent client from committing a crime/fraud that is reasonably certain to cause substantial financial injurythe client has used or is using the lawyers services o They did not use Worzaks services, so this is not available Potentially 1.6(b)(3) to prevent/mitigate/rectify substantial financial injury that is reasonably certain from a crime/fraud already committedthe client has used the lawyers services o They did not use Worzaks services, so this is not available o May/should/must Worzak take what he knows up the corporate ladder? The focus here is to protect the client which is the entity Lifecare. The corporation could suffer if the information is true, or gets out, etc. 1.13(b) commonly known as the reporting up obligation if a lawyer knows that an employee is engaged in action that is a violation of a legal obligation to the organization, or a violation of law that reasonably might be imputed to the organization, and that is likely to result in substantial injury to the organization, then the lawyer shall proceed as is reasonably necessary in the best of the organization. Unless the lawyer reasonably believes that it is not necessary in the best interest of the organization to do so, the lawyer shall refer the matter to higher authority in the organization, including, if warranted by the circumstances to the highest authority that can act on behalf of the organization as determined by applicable law.

60

Law of Lawyering Class Notes Talk to the General Counsel. If they do nothing, you have to keep trying. The GC or the VP for Compliance or whoever else is NOT the client. The client is the entity. You have to take it to the highest authority that can act on behalf of the organization (President, CEO, the Board). It is not a breach of the duty of confidentiality to report the information upward.

o If the Board of Directors refuses to act, does Worzak have the option or the duty to report out? The highest authority is NOT the client. The client is the entity. What can you do for the client? See Rule 1.13(c) (despite the lawyers effortsthe highest authorityfails to address in a timely and appropriate manner an action.clearly a violation of lawthat the violation is reasonably certain to result in substantial injury to the organization, then the lawyer may reveal information relating to the representation) PL: BUT SEE See Rule 1.13(d) (Paragraph (c) shall not apply with respect to information relating to a lawyers representation of an organization to investigate an alleged violation of law, or to defend the organization or an officer, employee or other constituent associated with the organization against a claim arising out of an alleged violation of law.) o So, since Worzak was assigned to investigate this, he cannot reveal it out. However, if Worzak simply came upon it, he could reveal it out (after reporting up and failing). See the Comments to Rule 1.13 for more information on the matter. o What are Worzaks options or obligations with respect to disclosure if the GC fires him now? He has taken the matter to the GC twice. See Rule 1.13(e) If a lawyer is discharged because of 1.13(b) (reporting up) or 1.13(c) (reporting out), or who withdraws under 61

Law of Lawyering Class Notes circumstances that require or permit the lawyer to take action under (b) or (c), shall proceed as the lawyer reasonably believes necessary to assure that the organizations highest authority is informed of the lawyers discharge or withdrawal, so you basically tell the Board you were fired and you can tell them why. o If Worzak is in a jurisdiction that allows for a cause of action for damages for retaliatory discharge of a lawyer due to a 1.13 situation, then Worzak can reveal the confidential information under 1.6(b)(5) to establish his claim for damages. o Suppose that Worzak is outside counsel. The head of the research department asks Worzak to represent him and the corporation in connection with a criminal investigation from the FDA application. What should you do? See 1.13(g) which takes you to Rule 1.7 and getting informed consent to represent both. The only additional issue of 1.13(g) is that the consenting official of the entity and the consenting individual cannot be the same person. (So, if you represent the CEO and the corporation, the corporations consent must come from someone other than the CEO.) Local Office, National Practice (pg. 670) (The following questions require application of Rule 5.5. In each case, be ready to discuss whether the actions are permitted by 5.5 Multijurisdictional Practice of Law.) o New York lawyers go to California to interview witnesses and review documents in preparation for a case that they will file in New York. This is allowable under 5.5(c)(2) (litigation) PL: Activities related to a potential case in which the lawyer will be authorized to appear. o A New York lawyer goes to California to take a deposition for use in a case pending in New York. This is allowable under 5.5(c)(2) PL: Activities related to a pending case in which the lawyer is authorized to appear.

62

Law of Lawyering Class Notes o A New York lawyer comes to California to interview witnesses for a case that the lawyer intends to file in California. PL: Activities related to a potential case in which the lawyer reasonably expects to be authorized to appear. To appear in court in a state that you are not licensed in, you have to get permission of the court (pro hac vice) and you generally need local counsel pursuant to 5.5(c)(1) PL: The need for local counsel when appearing pro hac vice arises not from 5.5 but from customary rules of pro hac admission. Note that 5.5 does not purport to regulate pro hac vice admission, That is left to the individual courts. o An associate licensed in New York comes to California to interview witnesses for a case that the associates supervisor (but not the associate) intends to file in California. This is allowable under 5.5(c)(2) or a person the lawyer is assisting

o New York lawyers come to California to represent a client in an arbitration proceeding. This is allowable under 5.5(c)(3) (arbitration)

o The lawyer in Local Office, National Practice leaves D.C. to negotiate a contract for a client in another state. This is allowable under 5.5(c)(4) (very broad and effectively includes transactional work) PL: And effectively excludes litigation and arbitration by its references to 5.5(c)(2) and (3) o The in-house lawyer with whom that lawyer negotiates is licensed in one state but lives and works in another. This is allowable under 5.5(d)(1) (work for the employer or organizational affiliate, basically made for in-house counsel for large organizations) o That same in-house lawyer represents the company in court in the state where the lawyer lives and works (but in which the lawyer is not licensed). PL: This would not be an authorized activity. The lawyer violated 5.5(a).

63

Law of Lawyering Class Notes o A Georgia lawyer with her main office in Columbus, Georgia establishes a satellite office in Phenix City, Alabama for the convenience of her Alabama clients. This lawyer is not doing temporary work and is not doing in-house work, but is rather establishing a systematic and continuous presence in a state where the lawyer is not authorized to practice. This is a violation of Rule 5.5, specifically under 5.5(b)(1) and (2). o 5.5(d)(2) federal law authorization must be given for this exception (such as patent law) I Dont Want to Pry, But (pg. 687) o Partner is concerned about a young lawyer who has not-so-conservative dress, makeup, hairstyle, way of carrying herself, way of talking, and a few occasionally visible tattoos and is apparently too thin o Rule 5.1 Responsibilities of Partners, Managers, and Supervisory Lawyers 5.1(a) Partners/Managers/Supervisors should take reasonable steps to ensure that all lawyers in the firm conform to the Rules of Professional Conduct 5.1(c) A lawyer shall be responsible for another lawyers violation of the Rules of Professional Conduct if the supervisor ordered or ratified the conduct or the partner, manager, or supervisor knew of the conduct when its consequences could be avoided or mitigated and failed to take reasonable remedial action o The major concern here is that of business and legal reputation more so than a violation of the RPC. When Sally Left Harry (pg. 704) o In the problem, When Sally Left Harry, [PL: The major point here is to show you how the Rule of Professional Conduct come into play in malpractice litigation, as evidence of the duty that may have been breached.] Adam did not seek in negotiation to obtain a portion of the increase in the value of the assets that Harry inherited. Putting aside any question of proving causation and damage, did Adam commit malpractice if: he did not know there was an argument that Sally was entitled to money? 64

Law of Lawyering Class Notes Yes, there is malpractice. The RPC help to prove this because they evidence the relationship and duty that was breached by duty (See Rule 1.1 and 1.3) he knew there might be such an argument but did not research the possibility before meeting with Harrys lawyer (assume he had time to do so)? Yes, there is malpractice. The RPC help to prove this as well. (See Rule 1.1 and 1.3) he knew there was such an argument but did not inject that issue into the negotiations because, in his judgment, to do so would have caused serious problems in being able to reach an agreement? This might be malpractice, but it does not seem like professional negligence since he knows all about the issue and is just exercising his own judgment. PL: But the failure to communicate could be -- This is also a breach of fiduciary duty as evidenced by Rule 1.4 and 1.2(a). o Just because the judgment turned wrong does not mean you should be civilly liable for malpractice. o Assume that Adam did not raise the issue and thereby breached a duty of competence to Sally. What will Sally have to show in order to recover in a cause of action for malpractice? How easy or difficult will that be? Duty, Breach, Causation, and Damages o Duty Duty of Competence o Breach Not knowing of and raising the issue o Causation This will be hard to show and prove (very speculative) o Damages This will be hard to show and prove (very speculative) PL: The problem is, what would have happened if Adam had raised it? There is no way to know how that would have played out in the negotiations without quite a bit of speculation.

65

Law of Lawyering Class Notes o What claim could Sally possibly bring based upon Adams partners representation of a general partnership in which Harry owned a substantial interest? Breach of duty of loyalty as evidenced by the RPC (Rule 1.7 for conflicts of interests) Duty Duty of Loyalty Breach Conflict of interest that was consentable and no informed consent was received or that was not consentable Causation and Damages will again be hard to show and prove (very speculative) What could Sally do if she could not prove damages? She could get her money back (disgorgement of fees) PL: See Section 37 of the Restatement of the Law Governing Lawyers Third: A lawyer engaging in clear and serious violation of duty to a client may be required to forfeit some or all of the lawyers compensation for the matter.

66

Vous aimerez peut-être aussi